2 - RoyalCanin.br

Transcrição

2 - RoyalCanin.br
Focus_SPECIAL_GI_port:01focuschatp0108FR 12/05/11 15:37 Página 1
Armadilhas em distúrbios
gastrintestinais no cão
Focus_SPECIAL_GI_port:01focuschatp0108FR 12/05/11 15:37 Página 2
A r m a d i l h a s
e m
d i s t ú r b i o s
g a s t r i n t e s t i n a i s
n o
c ã o
Sumário
Os autores
3
Introdução
7
1
Dez armadilhas comuns na obtenção do histórico
9
2
Exames clínicos para casos gastrintestinais
3
Exames complementares não instrumentais em distúrbios 23
gastrintestinais
4
Técnicas de diagnóstico por imagem e endoscopia do
sistema digestório: instruções de uso
31
5
Os principais erros cometidos no tratamento de
distúrbios gastrintestinais caninos
43
6
Perguntas frequentes e conceitos errados
51
Referências
62
2
15
Focus_SPECIAL_GI_port:01focuschatp0108FR 12/05/11 15:37 Página 3
A r m a d i l h a s
e m
d i s t ú r b i o s
g a s t r i n t e s t i n a i s
n o
c ã o
Os autores
Fileira de trás (da esquerda para a direita): Davide De Lorenzi, Valérie Freiche e Todd Tams.
Fileira da frente (da esquerda para a direita): Denise Elliott, Ewan McNeill e Carmem Rodriguez
Citology of the Dog and Cat (Atlas Colorido de
Citologia do Cão e Gato) de Baker e Lumsden.
Ele apresentou uma série de cursos de atualização
na Faculdade de Medicina Veterinária de Utrecht e
na Faculdade de Medicina Veterinária da
Universidade de Purdue em Indiana, EUA.
Em 2003, Davide foi conferencista/preletor na
Faculdade de Medicina Veterinária em Milão,
onde, através de esforço comum, foi responsável
pelo curso de patologia clínica; ele também
ministrou aulas sobre o mesmo assunto nas
Faculdades de Medicina Veterinária dentro das
Universidades de Pisa e Berne.
Como veterinário autônomo, ele trabalha
atualmente em Forli e Pádua, na Clínica Veterinária
São Marco, nas áreas de cirurgia geral, endoscopia
e citologia diagnóstica.
Davide De Lorenzi (Itália)
Davide De Lorenzi graduou-se em Medicina
Veterinária na Universidade de Bolonha em 1988.
Quatro anos depois, ele concluiu sua
especialização e recebeu título de Especialista em
Clínica Médica e Patologia de Animais de
Companhia na Faculdade de Medicina Veterinária
da Universidade de Pisa. Davide escreveu (como
único autor ou coautor) mais de 30 trabalhos e
comunicações — publicados em periódicos
italianos e internacionais — sobre temas
referentes à citologia diagnóstica, cirurgia e
endoscopia. Ele frequentemente ministra palestras
sobre esses assuntos em congressos veterinários.
Lorenzi também traduziu e publicou a versão
italiana do tratado intitulado Colour Atlas of
3
Focus_SPECIAL_GI_port:01focuschatp0108FR 12/05/11 15:37 Página 4
A r m a d i l h a s
e m
d i s t ú r b i o s
g a s t r i n t e s t i n a i s
n o
c ã o
trabalho na clínica Frégis, exclusivamente na área
de gastrenterologia. Valérie escreveu muitos
artigos e ministrou palestras sobre gastrenterologia em diversas conferências. Há muitos anos,
ela é membro do Grupo de Trabalho em Medicina
Interna (Working Group on Internal Medicine
[GEMI]) da Associação Francesa de Veterinários de
Pequenos Animais (French Association of
Companion Animal Vets [AFVAC]).
Em 2005, Davide obteve seu diploma na Faculdade
Europeia de Patologia Clínica Veterinária (European
College of Veterinary Clinical Pathology).
Denise Elliott (EUA)
Denise Elliott graduou-se na Universidade de
Melbourne com título de bacharel em Medicina
Veterinária em 1991. Depois de concluir estágio em
Clínica e Cirurgia de Pequenos Animais na
Universidade da Pensilvânia, Denise mudou-se
para a Universidade de Califórnia-Davis, onde
concluiu residência em Medicina de Pequenos
Animais, bolsa de estudos em Medicina Renal e
Hemodiálise e residência em Nutrição Clínica de
Pequenos Animais. Denise recebeu certificado de
especialista com a Faculdade Norte-Americana de
Medicina Interna Veterinária em 1996 e com a
Faculdade Norte-Americana de Nutrição
Veterinária em 2001. A Universidade de CalifórniaDavis concedeu à Denise o título de PhD em
Nutrição em 2001 por seu trabalho sobre Análise
de Impedância Bioelétrica de Múltipla Frequência
em Cães e Gatos Saudáveis. Atualmente, Denise é
diretora de Comunicações Científicas da Royal
Canin nos Estados Unidos.
Ewan McNeill (Reino Unido)
Ewan McNeill graduou-se pela Faculdade de
Medicina Veterinária da Universidade de Glasgow
em 1984. Esteve 5 anos trabalhando como clínico
geral em uma clínica, período durante o qual ele
estudou e obteve o Certificado em Radiologia
Veterinária. Em 1989, Ewan mudou-se para uma
clínica 100% de pequenos animais na cidade de
Nottingham até se concentrar na área de animais
domésticos; ele e sua esposa, também veterinária,
são os principais sócios dessa clínica no momento.
Atualmente, Ewan é presidente eleito da
Sociedade de Clínicos Veterinários (Society of
Practising Veterinary Surgeons). Além de seu
trabalho clínico, ele escreve para vários periódicos
profissionais sobre diversos tópicos e tem
ministrado
palestras
em
conferências
internacionais sobre assuntos referentes à gestão
veterinária. Ele também é responsável técnico de
uma importante universidade no Reino Unido e
uma empresa farmacêutica internacional.
Valérie Freiche (França)
Depois de se graduar na Escola Veterinária de
Alfort (Alfort National Veterinary School) em 1998,
a doutora Valérie Freiche permaneceu na faculdade
como membro do corpo docente, primeiramente
como estagiária, depois por mais dois anos como
professora assistente no Departamento de
Medicina. Durante esse período, ela abriu uma
clínica veterinária perto de Paris e se associou à
clínica Frégis, onde está atuando há 7 anos.
Valérie Freiche ainda é chefe da clínica de
gastrenterologia desenvolvida por ela dentro do
Departamento de Medicina da Escola Veterinária
de Alfort. Nos últimos 6 anos, ela também tem
Carmen Rodriguez (Espanha)
Carmen Rodriguez obteve título em Estudos
Veterinários da Universidade de Madri em 1985.
Desde então, Carmen é sócia do Centro Policlínico
Veterinário de Raspeig em San Vincente, província
de Alicante. A clínica foi fundada em 1949 por seu
pai, Manuel Isidro Rodriguez, e atualmente ela é
coproprietária desse centro policlínico.
4
Focus_SPECIAL_GI_port:01focuschatp0108FR 12/05/11 15:37 Página 5
A r m a d i l h a s
e m
d i s t ú r b i o s
g a s t r i n t e s t i n a i s
n o
c ã o
1982. Dr. Tams retornou a Los Angeles em 1984 e,
atualmente, é diretor médico dos VCA (de
Veterinary Centers of America, Centros
Veterinários da América) de Antech, uma empresa
que possui e opera 380 hospitais de pequenos
animais em todos os Estados Unidos. Dr. Tams
também atua nas áreas de medicina interna e
gastrenterologia no Hospital Veterinário da Parte
Ocidental de Los Angeles pertencente aos VCA,
uma clínica de 28 veterinários que oferece tanto
serviços de cuidados gerais como ampla gama de
serviços de especialidades. Todd foi nomeado
como ex-aluno com distinção da Faculdade de
Medicina Veterinária da Universidade do Estado de
Ohio em 2000. Além disso, Dr. Tams publicou dois
tratados: Endoscopia de Pequenos Animais
(Elsevier), segundo edição publicada em novembro
de 1998 e Manual de Gastrenterologia de
Pequenos Animais (Elsevier), segunda edição
publicada em 2003.
As áreas de interesse de Carmen envolvem as
técnicas de diagnóstico por imagem (em princípio,
radiologia e, mais recentemente, endoscopia) e os
exames laboratoriais, em particular nas áreas de
patologia gastrintestinal e urinária. Nos últimos 7
anos, ela seguiu seu interesse na área de etologia
clínica de cães e gatos e também é responsável
pelos pets exóticos tratados em seu hospital.
Carmen participou de testes de campo para
produtos nutricionais e outros procedimentos (em
particular, juntamente com seu marido, graduado
em estudos veterinários e medicina humana, ela
estuda sobre as dietas da Waltham® para cães
com deficiência renal causada por Leishmaniose).
Em 1996, ela trabalhou no Hospital Veterinário da
Universidade do Estado de Ohio nos Estados
Unidos, com revezamentos em disciplinas clínicas,
incluindo radiologia, ultrassonografia e
endoscopia. Ela tem ministrado palestras sobre
assuntos como diagnóstico radiológico, casos
clínicos em distúrbios digestivos, patologia do
comportamento e diagnóstico diferencial de
doenças em medicina interna; publicou esses e
outros assuntos, incluindo patologia da doença
urinária felina. Além disso, Carmen é membro do
Conselho Veterinário de Valência com
responsabilidade por questões relacionadas à
etologia de pets.
Todd Tams (EUA)
Dr. Todd Tams recebeu seu título de médico
veterinário pela Universidade do Estado de Ohio
em 1977 e, inicialmente, trabalhou em uma clínica
mista em Vermont de 1977-78. Ele, então, concluiu
estágio em pequenos animais no Hospital
Veterinário da Parte Ocidental de Los Angeles,
Califórnia em 1978-79, seguido por residência em
medicina interna na Universidade do Estado do
Colorado de 1979-81. Todd foi membro da equipe
de residentes do Hospital Veterinário Angell
Memorial em Boston de 1981-84 e obteve
certificado de especialista em medicina interna em
5
Focus_SPECIAL_GI_port:01focuschatp0108FR 12/05/11 15:37 Página 6
A r m a d i l h a s
e m
d i s t ú r b i o s
6
g a s t r i n t e s t i n a i s
n o
c ã o
Focus_SPECIAL_GI_port:01focuschatp0108FR 12/05/11 15:37 Página 7
A r m a d i l h a s
e m
d i s t ú r b i o s
g a s t r i n t e s t i n a i s
n o
c ã o
Introdução
Os distúrbios gastrintestinais estão entre as queixas mais comuns na clínica geral, trazendo problemas reais
aos veterinários tanto em termos de diagnóstico (p. ex., pelo histórico vago fornecido pelo proprietário, pelos
sintomas inespecíficos e pela ampla variedade de exames complementares, algumas vezes confusos) como
em relação ao tratamento (p. ex., escolha de antibióticos, duração do tratamento nutricional, etc.).
A finalidade dessa edição da revista Focus Special é listar — de forma lógica, racional e coerente — as
principais armadilhas envolvidas nesses distúrbios, dissipar alguns dos conceitos errados e tentar responder
a perguntas difíceis, tais como: “qual o significado da citologia retal?”, “as radiografias contrastadas são
mais proveitosas que a ultrassonografia?”, “qual o melhor tratamento para doença inflamatória intestinal?”
e “qual a importância da gordura em uma dieta destinada ao tratamento de diarreia crônica?”
Esta é a primeira vez que a revista Focus foi escrita por uma equipe internacional de seis veterinários: o
primeiro trabalho realizado pelos europeus Davide De Lorenzi (Itália), Valérie Freiche (França), Ewan McNeill
(Reino Unido) e Carmen Rodriguez (Espanha) foi expandido com as contribuições de Denise Elliott (Austrália)
e Todd Tams (Estados Unidos). Gostaríamos de expressar nossa sincera gratidão a esses autores por sua real
contribuição a esse complexo assunto.
Dr. Vincent C. Biourge
Médico veterinário (DVM, PhD)
Diplomado por ACVN (American College of Veterinary Nutrition) e
ECVCN (European College of Veterinary and Comparative Nutrition)
Centro de Pesquisas da Royal Canin
França
7
Focus_SPECIAL_GI_port:01focuschatp0108FR 12/05/11 15:37 Página 8
A r m a d i l h a s
e m
d i s t ú r b i o s
8
g a s t r i n t e s t i n a i s
n o
c ã o
Focus_SPECIAL_GI_port:01focuschatp0108FR 12/05/11 15:37 Página 9
A r m a d i l h a s
e m
d i s t ú r b i o s
g a s t r i n t e s t i n a i s
n o
c ã o
1. Dez armadilhas comuns na
obtenção do histórico
> Resumo
*
A seguir, estão expostas dez das armadilhas mais comuns reservadas para o clínico na obtenção do
histórico de algum caso durante a investigação de problema gastrintestinal
1. Não compreender a queixa principal do proprietário
2. Não levar em conta a identificação do paciente
3. Não levar em consideração o histórico veterinário prévio
4. Não obter informações básicas suficientes na anamnese
5. Não dar a devida atenção à dieta
6. Ser induzido pelo proprietário
7. Não trazer à tona detalhes suficientes pelo proprietário
8. Não manter uma mente aberta quanto à causa inicial
9. Não distribuir tempo/método suficiente para obtenção do histórico
10. Deixar escapar os casos não habituais
histórico na pressa, a esmo ou parcialmente pode
levar o veterinário a seguir um caminho diagnóstico
errado com consequências potencialmente
desastrosas. Vale à pena considerar os erros mais
comuns cometidos na obtenção do histórico para
permitir um relato preciso feito pelo proprietário.
Introdução
Os distúrbios gastrintestinais constituem uma das
causas mais comuns que levam um cão ao
consultório do clínico de pequenos animais, mas
também representam uma das áreas mais
frequentes em que o diagnóstico errado da
condição ou a resolução incompleta do problema
pode resultar em insatisfação do proprietário,
frustração do veterinário ou, até mesmo, extenso
sofrimento do animal. A investigação satisfatória
de algum problema gastrentérico deve começar
obrigatoriamente com excelente obtenção do
histórico; com a abordagem sistemática na
anamnese, bem como com a anotação precisa e
compreensão da resposta do proprietário, o clínico
pode estabelecer um fundamento sólido e seguro
para a avaliação e o diagnóstico exatos do
problema. Com muita frequência, a obtenção do
1/ Não compreender a
queixa principal do
proprietário
Os veterinários são frequentemente culpados em admitir
que compreendem a natureza da queixa principal do
proprietário apenas com o mais breve dos diálogos no
início da consulta; por exemplo, o proprietário pode se
queixar de que seu cão está “mal e algumas vezes tem
diarreia”, fazendo com que o veterinário suponha que o
pet tem enterite, enquanto, na verdade, a diarreia pode
9
Focus_SPECIAL_GI_port:01focuschatp0108FR 12/05/11 15:37 Página 10
e m
d i s t ú r b i o s
g a s t r i n t e s t i n a i s
n o
c ã o
© Royal Canin
A r m a d i l h a s
Figura 2. Nem todo cão Boxer com
diarreia sofre de colite ulcerativa
histiocítica.
Figura 1. O histórico deve ser obtido
antes do exame clínico.
É extremamente fácil admitir a presença de alguma
doença associada à raça pelo fato de o cão pertencer à
determinada raça; p. ex., admitir que todo cão Boxer com
diarreia deva ter colite É igualmente importante não se
esquecer do óbvio — seria imprudente não incluir a
possibilidade, ou até mesmo a probabilidade, de
supercrescimento bacteriano do intestino delgado em
uma lista de diagnósticos diferenciais para um cão Pastor
Alemão com diarreia (Hall, 2005). Do mesmo modo, pode
ser surpreendentemente fácil negligenciar ou passar
despercebido pela possibilidade de piometrite em cadela
de meia-idade que se apresenta com vômito e letargia a
menos que alguém leve um instante para confirmar se o
animal é ou não castrado.
ser um fator secundário de alguma doença extraintestinal.
Portanto, é essencial determinar o que exatamente os
proprietários querem dizer quando eles descrevem o(s)
principal(is) sintoma(s) apresentado(s) pelo seu pet; é
preciso fazer um questionamento meticuloso para
identificar a manifestação exata do que preocupa o
proprietário. Além disso, deve-se estar sensível ao fato de
que, durante a avaliação, pode ficar evidente a presença
de algum outro problema mais urgente, mas possível e
totalmente separado daquilo que o proprietário descreve
— no entanto, ainda é prudente identificar o que o
proprietário nota e detecta como o principal problema
para evitar falha de comunicação em algum ponto futuro
durante o tratamento do caso (Radford, 2002).
3/ Não levar em
consideração o histórico
médico prévio
A confiança em si mesmo frequentemente pode fazer com
que o veterinário tenha uma “visão em túnel”, ou seja,
restrita (ou, até mesmo, a falta de percepção) e admita
que um paciente não tenha tido sintomas semelhantes no
passado, simplesmente porque o proprietário não
forneceu essa informação voluntariamente. De fato, é
2/ Não levar em conta a
identificação do paciente
10
Focus_SPECIAL_GI_port:01focuschatp0108FR 12/05/11 15:37 Página 11
A r m a d i l h a s
e m
d i s t ú r b i o s
g a s t r i n t e s t i n a i s
n o
c ã o
paciente tem vida errante (ou seja, se ele perambula
livremente) dentro de seu ambiente todos os dias e pode
habitualmente ingerir plantas domésticas, ou se o animal
é mantido como cão de guarda e pode ter livre acesso a
material estranho, ou se algum outro pet da casa também
teve sinais de distúrbio gastrintestinal há pouco tempo.
muito comum que o clínico deduza que o problema seja de
natureza aguda, enquanto, na verdade, ele pode ser bem
crônico, simplesmente pelo fato de o proprietário não ter
divulgado essa informação ou não ter percebido que
episódios de doença ocorridos há alguns meses ou até
mesmo anos podem estar relacionados ao quadro. A
definição da natureza aguda ou crônica da doença é uma
etapa importante na avaliação diagnóstica de algum caso.
Também é essencial obter as carteirinhas de vacinação e
vermifugação, e não assumir simplesmente que as
recordações do proprietário, que podem ser
assustadoramente vagas, sejam totalmente precisas.
5/ Não dar a devida atenção
à dieta
A dieta e os distúrbios gastrintestinais frequentemente
andam lado a lado. Por essa razão, é essencial gastar
algum tempo extraindo o histórico nutricional completo
dos proprietários, fazendo com que eles compilem um
“diário alimentar” de seu pet, se necessário (Figura 4).
Em particular, vale à pena considerar três pontos:
• A possibilidade de que o proprietário deixe de
mencionar, sem intenção, certo componente da dieta do
animal, componente este que pode ser um indício real
para algum problema intestinal — o exemplo clássico é o
oferecimento ocasional de leite ao cão, resultando em
diarreia por intolerância à lactose.
• A possibilidade de que o proprietário deixe de relatar de
forma voluntária a troca recente de um dos ingredientes
(fornecido constantemente por muitos meses) da dieta do
cão.
• O risco de admitir que a dieta deva ser o pivô do
problema — pode ser que a dieta seja totalmente
irrelevante ao caso.
4/ Não obter informações
básicas suficientes na
anamnese
Embora possa parecer trabalhoso gastar tempo sondando
o histórico do paciente, isso pode ser uma parte essencial
para obtenção de possíveis indícios quanto à origem de
algum problema (Figuras 1 e 3). Dessa forma, pode ser
extremamente útil obter detalhes sobre o modo de vida do
paciente; por exemplo, a casinha do animal, a
disponibilidade de petiscos e itens não alimentares, bem
como a presença de outros animais na casa — e se eles
ou alguma pessoa que têm contato com o paciente estão
doentes. Assim, o clínico pode tirar proveito de saber se o
6/ Ser induzido pelo
proprietário
© Valérie Freiche
Pode ser muito fácil ser induzido — ou induzido ao erro —
pelo proprietário em relação a algum diagnóstico, seja
porque o proprietário tem uma ideia fixa de que “o cão foi
intoxicado ou envenenado” ou, para dar um exemplo um
pouco mais específico, seja pelo fato de o problema ter
começado “depois de o cão retornar de um passeio, pois
ele deve ter apanhado alguma coisa quando estava na
rua”. Por outro lado, é totalmente possível que o clínico
induza o proprietário a algum diagnóstico, que foi
assumido logo que o animal entrou na sala de exame.
Esse erro pode ser confundido pelo uso de “questões
Figura 3. Antes de tocar no cão, o
clínico deve dedicar, pelo menos, 5
minutos (até 20 minutos em casos de
encaminhamento) para obter o
histórico
11
Focus_SPECIAL_GI_port:01focuschatp0108FR 12/05/11 15:37 Página 12
A r m a d i l h a s
e m
d i s t ú r b i o s
g a s t r i n t e s t i n a i s
n o
c ã o
de anorexia ou perda de peso podem ser atribuídos a um
problema gástrico primário mesmo na ausência de
vômito.
fechadas” (ou seja, perguntas que tendem a ser
respondidas por “sim”/”não”), o que pode levar o
proprietário a fornecer respostas equivocadas sem
intenção.
9/ Não distribuir
tempo/método suficiente para
7/ Não trazer à tona detalhes obtenção do histórico
suficientes pelo proprietário
Pode ser uma falsa economia de tempo tentar obter todos
os detalhes necessários em uma breve consulta com o
proprietário. Isso talvez seja particularmente verdadeiro
em casos crônicos ou em situações em que se busca uma
segunda opinião, pois pode ser quase irresistivelmente
tentador embarcar em um curso de investigação ou até
mesmo em algum tratamento sem a obtenção de histórico
válido e detalhado. Se necessário, deve ficar claro para o
proprietário que é melhor, em benefício de todos,
reagendar a consulta para conferir tempo suficiente ao
exame e garantir a aquisição de bases sólidas, sem
pressa.
Além disso, um clínico ocupado ou inexperiente pode não
usar uma abordagem metódica para obter o histórico;
checklists escritos talvez ajudem a garantir que nenhum
marcador diagnóstico potencialmente importante seja
negligenciado por uma má anamnese.
É essencial obter relatos detalhados do proprietário sobre
o problema de seu cão. Não basta, por exemplo, aceitar
um histórico composto por uma única palavra, como
“vômito”; é imperativo que uma descrição mais completa
seja obtida (DeNovo e Jenkins, 1998). O proprietário pode
notar que o cão vomita logo após comer, mas uma
abordagem metódica na anamnese pode deduzir que
esse animal, na verdade, está regurgitando alimento, ou
tem disfagia (dificuldade de deglutição), ou tem ânsia de
vômito por algum problema de traqueia. A apuração dos
fatos, tais como quanto tempo depois da refeição o pet
vomita, se o vômito é de natureza impulsora ou não
(vômito em jato ou projétil), a coloração e a consistência
do vômito, a presença ou não de sangue (em caso
afirmativo, verificar os materiais, como o aspecto e
volume do sangue), pode ajudar a evitar afirmações
potencialmente equivocadas. Do mesmo modo, nos casos
em que o proprietário descreve “diarreia”, é fundamental
identificar completamente as características do problema
— por exemplo, o volume, a frequência e a consistência
das fezes; a presença ou ausência de sangue ou muco; a
existência ou não de qualquer sinal de tenesmo ou
disquezia (Hall, 1998).
10/ Deixar escapar os casos
não habituais
Pode ser tentador assumir que toda doença se enquadra
perfeitamente aos casos arquivados e, nessa
circunstância, os clínicos podem cair na armadilha de
distorcer todos os detalhes do histórico (e, possivelmente,
até mesmo dos achados clínicos) para se adequar ao seu
diagnóstico preferido. É essencial ter uma mente aberta
para algum problema intratável ou recorrente, com a
disposição de rever o histórico, conforme e quando
necessário. Esteja atento também ao inesperado; por
exemplo, com o crescimento maciço dos pets que
acompanham seus proprietários de férias, as doenças que
previamente eram consideradas como tropicais ou
estrangeiras devem, hoje em dia, ser consideradas como
de origem exótica. Em particular, os pets que são
provenientes das regiões mais temperadas e visitam um
8/ Não manter a mente
aberta quanto à causa inicial
Também é muito fácil esquecer que nem todas as doenças
de natureza aparentemente gastrintestinal, por conta do
principal sinal apresentado (tais como vômito ou diarreia),
necessariamente se originam do sistema gastrintestinal;
os problemas renais e hepáticos são exemplos clássicos
disso. Da mesma forma, pode passar despercebido o fato
de que os animais sem vômito ou diarreia como o principal
sinal apresentado tenham problemas que se originam de
distúrbios gastrintestinais — por exemplo, os sintomas
12
Focus_SPECIAL_GI_port:01focuschatp0108FR 12/05/11 15:37 Página 13
A r m a d i l h a s
e m
d i s t ú r b i o s
clima mais quente podem contrair doenças, que se
manifestam somente em algum momento depois de o
animal ter retornado para casa; sendo assim, pode ser
sábio que o clínico acrescente uma pergunta sobre
viagens durante a obtenção do histórico.
g a s t r i n t e s t i n a i s
n o
c ã o
diferentes aspectos dos problemas de seu pet; além disso,
é prudente reservar tempo suficiente para obtenção de
histórico completo e preciso antes de embarcar em novas
investigações diagnósticas ou iniciar os regimes
terapêuticos.
Conclusão
Figura 4. Se o
cão vive na rua
ou tem acesso
à rua, é
impossível
predizer o que
ele pode
comer.
Para uma investigação bem-sucedida de distúrbios
gastrintestinais, é essencial que o veterinário adote uma
abordagem sistemática para obtenção do histórico; os
checklists podem ser muito valiosos para ajudar em uma
abordagem metódica. Sempre é sábio ter em mente a
identificação do paciente e garantir que todos os detalhes
médicos e ambientais sejam conhecidos. O proprietário
deve ser meticulosamente questionado sobre os
© Royal Canin
> Perguntas a Denise Elliott
Perguntas a serem feitas sobre o histórico nutricional
- Comercial versus caseira,
- Ad libitum (ou seja, à vontade) versus refeição
programada versus alimentação em horários restritos
- Crua versus cozida,
• Quantas refeições são fornecidas por dia?
- Seca versus úmida versus semiúmida.
• Quem alimenta o animal?
• Se a comida fornecida for caseira, obter uma lista
completa de todos os ingredientes (inclusive os
suplementos vitamínico-minerais), as quantidades exatas,
o método de preparo/cozimento e armazenagem).
- Muitas vezes, o principal tratador do animal não é a
pessoa que leva o cão ao veterinário.
•Que tipo de comida é fornecido ao cão?
Denise Elliott
BVSc (Bachelor of Veterinary
Science), PhD, diplomada por
ACVIM (American College of
Veterinary Internal Medicine)
ACVN (American College of
Veterinary Nutrition), diretora das
comunicações científicas
da Royal Canin nos Estados Unidos
• Se o alimento for comercial, qual o fabricante e a marca
da ração?
• Há quanto tempo o cão está recebendo essa dieta?
• Quando a dieta do cão foi trocada pela última vez?
• Com que frequência a dieta do cão é trocada?
• Qual a quantidade de alimento recebido pelo cão em
cada refeição?
- Se o alimento for seco, verificar o método de medida, ou
seja, qual o tamanho do copo-medida?
• Qual é o método de alimentação?
13
• O pet recebe algum suplemento nutricional, alimento
humano, petisco ou lanche? Em caso afirmativo, qual o
tipo, a quantidade e com que frequência?
?
• Qual é a dinâmica da família? Existem crianças, idosos
ou outras pessoas que vivem na casa e podem alimentar o
cão inadvertidamente?
• Existem outros pets na casa? Se sim, quais os tipos de
pets (gatos, pássaros, etc.)? Como esses pets são
alimentados? O cão tem acesso ao alimento desses pets?
• O cão tem acesso livre (ou seja, não supervisionado) à
rua? Em caso afirmativo, há vizinhos que podem
inadvertidamente alimentar o cão? Ou o cão pode ter
acesso ao alimento dos pets da vizinhança?
Focus_SPECIAL_GI_port:01focuschatp0108FR 12/05/11 15:37 Página 14
A r m a d i l h a s
e m
d i s t ú r b i o s
g a s t r i n t e s t i n a i s
n o
c ã o
Sistema de escore fecal para cães
1
Fezes muito
amolecidas,
diarreia
2
Mistura de fezes
amolecidas,
principalmente
malformadas
3
Fezes formadas,
porém muito
moles
4
Fezes formadas
e mais secas,
porém não duras
5
Fezes formadas,
secas e duras
14
Focus_SPECIAL_GI_port:01focuschatp0108FR 12/05/11 15:37 Página 15
A r m a d i l h a s
e m
d i s t ú r b i o s
g a s t r i n t e s t i n a i s
n o
c ã o
2. Exame clínico para casos
gastrintestinais
*
> Resumo
Tendo coletado todas as informações possíveis a respeito do histórico de caso do paciente, o
veterinário deve passar para o exame clínico e considerar a realização de exames complementares
não instrumentais apropriados. Um exame consciencioso e meticuloso do paciente possibilitará a
obtenção da máxima quantidade possível de informações pelo clínico, o que, por sua vez, torna mais
fácil a escolha de exames complementares adequados, ao mesmo tempo em que se evita gasto
excessivo em termos de tempo e dinheiro. Essa abordagem lógica e racional maximizará as chances
de alcançar o diagnóstico correto e, acima de tudo, deve ajudar a evitar a “visão em túnel” (ou seja,
restrita).
1/ Avaliação diagnóstica
minuciosa
O clínico deve conduzir toda avaliação clínica com
uma abordagem metódica e padronizada composta
por 3 fases.
A) Estado de Alerta
Apesar de compilar cuidadosamente o histórico do
caso com o proprietário, o clínico deve garantir que a
primeira coisa a ser observada, até mesmo antes de
tocar no paciente ou colocá-lo na mesa de exame,
seja a atitude ou o estado de alerta do animal. Isso
envolve a observação da resposta do animal a
estímulos, como ele se desloca, sua postura corporal
e sua tendência a explorar seus arredores.
Os sinais sugestivos e significativos podem
compreender um ou mais dos itens expostos a
seguir:
•Comportamento sonolento e letárgico, que pode
ser observado em distúrbios metabólicos com
influência sobre o sistema nervoso central (SNC),
tais como hepatopatia grave (desvios
portossistêmicos, etc.). Nesse caso, também se
pode observar hiperatividade associada com a
síndrome de pica (ingestão de objetos). Isso se deve
aos metabólitos semelhantes a neurotransmissores
e produzidos como resultado da deficiência hepática
(Cauzinille, Bouvy, 2003). Também pode haver falta
de resposta a estímulos em hipoproteinemias
graves.
•Movimento lento e postura arqueada, que podem
indicar a presença de dor, causada por problemas
como corpos estranhos localizados ou presos em
algum ponto dentro do trato intestinal, pancreatite,
peritonite, pielonefrite, enterite viral, esofagite,
duodenite ou inflamação grave do ceco.
•Fraqueza muscular, que pode ser atribuída a
hipoadrenocorticismo, miastenia grave, caquexia,
hipocalemia (secundária a vômito ou obstruções
intestinais graves) ou doença neuromuscular difusa.
•Ataxia, déficit proprioceptivo e fraqueza dos
membros, conforme são observados em distúrbios
do SNC ou doença neuromuscular difusa.
•Mialgia (dor muscular) em casos de polimiosite
(que pode se manifestar sob a forma de
megaesôfago).
15
Focus_SPECIAL_GI_port:01focuschatp0108FR 12/05/11 15:37 Página 16
A r m a d i l h a s
e m
d i s t ú r b i o s
g a s t r i n t e s t i n a i s
n o
c ã o
Abordagem metodológica e padronizada em 3 estágios
Atitude do cão
• Estado de alerta
• Músculos
• Comportamento
Observação física
• Pele e pelagem
• Mucosas
• Ânus
•
•
•
•
•
•
Exame clínico
•Artralgia (dor articular) e claudicação, observadas
em casos de lúpus eritematoso sistêmico (LES) ou
outros distúrbios imunológicos indutores de artrite;
notar que essas doenças também podem causar
distúrbios intestinais inflamatórios ou hepatite ativa
crônica (Jergens, 1999) (Magne, 2000).
•Desconforto oral em casos de intolerância
intestinal ou pica, refluxo gastresofágico, ou dor
gastroduodenal.
•Agressão ou irritabilidade em casos de apetite
excessivo (por má digestão ou má-absorção) ou
decorrente de desconforto abdominal.
Auscultação cardíaca
Auscultação pulmonar
Cavidade oral
Palpação abdominal
Ânus e porção distal do reto
Coleta de fezes
• Elasticidade insatisfatória da pele em casos de
desidratação.
• Mucosas (a) congestas com preenchimento
capilar lento em casos de desidratação ou má
perfusão tecidual, (b) cinzas e pouco oxigenadas por
dor, choque ou enterotoxemia, (c) pálidas na
presença de distúrbio hemorrágico agudo ou crônico
e (d) ictéricas em doença do fígado, pâncreas ou
vesícula biliar.
• Aparência anormal do ânus, que pode ser
atribuída a problemas como fístulas perianais
(Figura 1) ou carcinoma anorretal (Figura 3). Esses
problemas podem estar associados com disquezia,
incontinência ou constipação, dependendo da
evolução da doença.
B) Observação Física
O veterinário, então, deve considerar o estado físico
do animal: isso deve incluir avaliação do (1) aspecto
da pelagem, (2) da qualidade (incluindo as áreas
cutâneas queratinizadas) e elasticidade da pele, (3)
da coloração, umidade e tempo de preenchimento
capilar das mucosas, bem como (4) da aparência da
região anal. Isso pode revelar o seguinte:
• Má qualidade da pelagem ou da pele, que pode
ser observada em casos de má-absorção e má
digestão avançados; também pode haver um nariz
seco e rachado. Caso se observe a presença de
outros sintomas cutâneos, considerar possíveis
causas
infecciosas/parasitárias,
como
leishmaniose, capaz de causar dermatose
esfoliativa ou granulomas.
C) Exame Clínico
Na sequência, o veterinário deve realizar a
auscultação cardíaca e pulmonar, bem como a
palpação da região cervical. Isso pode revelar:
•Sopros sistólicos funcionais (p. ex., em casos de
anemia grave).
•Bradicardia e pulso femoral débil (p. ex., em casos
de hipoadrenocorticismo), taquicardia e pulso fraco
(enterite infecciosa com choque séptico), ou
taquicardia com déficit de pulso (dilatação gástrica).
•Ruídos respiratórios na presença de disfagia
indutora de aspiração de material alimentar.
•Ruído (pela presença de líquido ou ar) quando a
16
Focus_SPECIAL_GI_port:01focuschatp0108FR 12/05/11 15:37 Página 17
A r m a d i l h a s
e m
d i s t ú r b i o s
Figura 1. Fístula perianal em
cão Pastor Alemão de 5
anos de idade.
© Valérie Freiche
n o
c ã o
Candidíase). Esses casos também exibirão sialorreia
(Lecoindre, 2004).
•Halitose provocada por alimentos fermentados ou
ossos presos no esôfago, doença gastrintestinal
crônica, neoplasia ou doença periodontal.
Isso pode ser sucedido por auscultação, percussão e
palpação abdominais para verificar a presença de
borborigmos, gases dentro do intestino, ascites ou
abdome retraído. Os sinais significativos podem
incluir um ou mais dos itens a seguir:
• Borborigmos ou ruídos intestinais causados pelo
deslocamento de líquido e gás no intestino. Isso
pode ser observado em condições que promovem e
favorecem a fermentação bacteriana por máabsorção de carboidratos e proteínas (Tams, 2003)
(inflamação crônica do intestino delgado,
supercrescimento bacteriano do intestino delgado).
Os borborigmos podem ser normais em casos de
aerofagia se o animal for um devorador voraz (ou
© Valérie Freiche
© Carmen Rodriguez
porção cervical do esôfago ou a região da entrada
torácica são palpadas (p. ex., dilatação esofágica),
bem como deglutição exagerada ou mais frequente
(p. ex., arco aórtico direito persistente, megaesôfago
ou corpo estranho esofágico). Notar que é essencial
levar o estado de inquietação do paciente em conta
e que, se o cão estiver apreensivo, é porque ele está
na cirurgia; a disposição nervosa do animal pode
indicar que esse sinal é superinterpretado!
•Dispneia, que pode ser sugestiva de problema
esofágico (p. ex., corpo estranho, hérnia hiatal)
(Gualtieri, 2004).
Isso deve ser acompanhado por exame da cavidade
oral. Isso pode revelar:
•Sialorreia (salivação excessiva) por dor apesar da
presença de deglutição (odinofagia), causada por
corpo estranho (como agulhas ou ossos) ou
esofagite (Lecoindre, 2004).
•Ulceração oral atribuída à uremia ou estomatite
oportunista em animais imunodeprimidos (p. ex.,
g a s t r i n t e s t i n a i s
Figura 2. O exame clínico do Pastor
Alemão da figura 1 revelou a
presença de micronódulos no
esfíncter anal externo.
Figura 3. Demonstração de inúmeros
pólipos (carcinoma anorretal) por meio de
palpação retal em cão da raça West
Highland White Terrier.
17
Focus_SPECIAL_GI_port:01focuschatp0108FR 12/05/11 15:37 Página 18
A r m a d i l h a s
e m
d i s t ú r b i o s
seja, comer rapidamente) ou se ele tiver ingerido
alimento produtor de gás (leguminosas, repolho,
etc.). Nesse caso, também se pode notar a presença
de flatulência, produzindo odor desagradável. Notar
que 99% dos gases gastrintestinais, compostos de
oxigênio, nitrogênio, dióxido de carbono, hidrogênio
e metano, são inodoros (Tams, 2003). Os
borborigmos também podem ser constatados se
houver algum distúrbio funcional (p. ex., cólon
irritável (Guilford, 2002).
•Intestinos preenchidos por gases em virtude de
redução da motilidade intestinal (p. ex., doença
grave da parede intestinal, animais idosos com
espondiloartrose capaz de afetar a inervação da
musculatura digestiva, obstrução (Guilford, 2002)).
• Abdome pendular por ascite, causada por
hipoproteinemias intensas (secundárias à doença
crônica grave do intestino delgado ou hepatopatia
grave).
• Abdome retraído na presença de dor (p. ex., por
enterite viral, pancreatite, peritonite, pielonefrite,
hepatopatia congestiva, corpos estranhos alojados
em parte do trato digestório, ou intussuscepção).
Ocasionalmente, a palpação meticulosa permitirá a
localização da dor (porção abdominal cranial
esquerda em doença hepática, cranial direita em
pancreatite, média dorsal em inflamação do ceco,
cranial dorsal em intussuscepção). A palpação das
alças intestinais situadas na parte cranial do
abdome pode se tornar mais fácil, fazendo com que
o animal se mantenha sobre os membros traseiros,
com o proprietário segurando os membros
dianteiros.
g a s t r i n t e s t i n a i s
n o
c ã o
Notar que o procedimento de tranquilização e
consequente relaxamento da parede abdominal
pode ajudar na palpação de órgãos do abdome; isso
pode ser particularmente útil ao se examinar
neoplasia gástrica ou linfonodos infartados sob
suspeita
(Figura 4).
Isso deve ser acompanhado por palpação de 360° do
ânus e da porção distal do reto com os dedos; esse
exame pode identificar:
•Dor e sensibilidade da mucosa retal (p. ex.,
causadas por fístula perianal, doença dos sacos
anais, proctite, hérnia perineal, carcinoma
anorretal).
•Textura anormal da mucosa retal (p. ex.,
endurecimento por tecido granulomatoso nos
estágios iniciais de fístulas perianais) e pólipos
anorretais (Figura 2).
• Estenose anorretal ou colorretal (p. ex., em casos
mais avançados de fístula perianal, doença
infiltrativa como carcinoma retal, e formação de
tecido cicatricial associada com cirurgia do colo
uterino (Tams, 2003) ou episódio prévio de distocia).
Durante essa parte do exame, as fezes devem ser
coletadas para inspeção direta e avaliação de sua
cor, odor e textura, o que pode sugerir o seguinte
(Figura 5 a 8):
•Coloração anormal; amarelo-limão (em
insuficiência pancreática exócrina [IPE]), de ocre a
castanho escuro ou laranja (em enterite inflamatória
crônica), esverdeado escuro (na maioria dos casos
© Valérie Freiche
Figura 4. Esse cão magro da
raça Chow Chow,
encaminhado por causa de
vômito há 2 meses, tinha, na
verdade, linfonodos
aumentados em função de
linfoma multicêntrico.
18
Focus_SPECIAL_GI_port:01focuschatp0108FR 12/05/11 15:37 Página 19
A r m a d i l h a s
e m
d i s t ú r b i o s
agudos de supercrescimento bacteriano), manchas
negras (na presença de sangramento intermitente do
trato digestório proximal) ou quase totalmente
negras (em casos de melena em que há hemorragia
abundante do trato gastrintestinal proximal).
•Odores fecais; por exemplo, odor pútrido em
fermentação bacteriana, ou odor ácido em casos de
má-absorção, ou quase inodoro na presença de má
digestão.
•Presença de muco e sangue fresco, que podem
estar bem-misturados nas fezes (se originarem do
ceco e da porção proximal do cólon) ou cobrindo as
fezes (se atribuídos a proctite ou pólipos retais).
•Textura anormal, por exemplo, gordurosa (em
alguns casos de IPE), pegajosa (em melena), líquida
(em diarreias secretórias agudas, embora também
ocorra em alguns casos “intratáveis” crônicos);
•Presença de material de origem não alimentar,
como plásticos, madeira, etc., o que pode indicar
pica.
2/ Armadilhas frequentes
no exame clínico
n o
c ã o
metodologia
Examinar apenas os órgãos associados com os
sintomas clínicos descritos pelo proprietário pode
significar a falha na obtenção do diagnóstico correto.
Por exemplo, se o clínico ignorar a região esofágica
em cão com queixa de vômito, isso pode precipitar
uma falha na identificação de paciente com
megaesôfago.
Também é muito fácil pular parte do exame se o
proprietário estiver conversando com o veterinário
ou distraindo-o de alguma forma.
É fácil ser tentado a prosseguir com os exames
complementares antes de concluir todo o exame
clínico. Isso pode significar a realização de mais
exames do que realmente é necessário ou a omissão
de exames essenciais. Por exemplo, em caso de
paciente com vômito intermitente há 10 dias, o
clínico pode embarcar em um estudo radiográfico
com bário (meio de contraste) na suspeita de corpo
estranho sem efetuar exame clínico completo. Se o
clínico reservar um tempo para exame das fezes, o
aspecto desse material pode induzir a exames mais
apropriados, como imunorreatividade semelhante à
da tripsina e estudo endoscópico para pesquisar a
possibilidade de deficiência pancreática ou
gastrenterite linfoplasmocitária.
© Carmen Rodriguez
A) Realizar exame clínico sem
g a s t r i n t e s t i n a i s
Figura 5. Fezes de cão
que sofre de
insuficiência
pancreática exócrina.
Notar a coloração
amarelada brilhante em
virtude da presença de
gordura.
Figura 6. Fezes de cão
que sofre de colite
linfoplasmocitária
erosiva crônica leve.
Notar as estrias
avermelhadas e a
presença de muco.
19
Figura 7. Fezes de cão
que sofre de
gastrenterite
eosinofílica, com
aparência escura pela
presença de sangue
digerido.
Figura 8. Fezes de cão
que sofre de
gastrenterocolite
linfocítica-plasmocitária
moderada, com a
presença de muco e
pedaços de madeira
ingeridos por pica.
Focus_SPECIAL_GI_port:01focuschatp0108FR 12/05/11 15:37 Página 20
A r m a d i l h a s
e m
d i s t ú r b i o s
g a s t r i n t e s t i n a i s
n o
c ã o
B) Interpretar os sinais clínicos
de forma isolada
D) Presumir que o exame na sala
de consulta é suficiente
Vômito, diarreia ou perda de peso não são sinais
isolados. Na maioria dos casos, esses sinais
costumam vir acompanhados por outras evidências
clínicas, que devem ser interpretadas como um todo.
É essencial não focar apenas em um único sinal
clínico; no entanto, a importância de se adotar uma
abordagem holística não pode ser superenfatizada.
Talvez haja necessidade de internação do paciente
para inspeção direta se os sintomas descritos pelo
proprietário não estiverem claros ou quando o
animal vive na rua e se parecer desejável ou
conveniente a observação de questões como o tipo
de vômito ou como o animal defeca.
C) Pressupor que algum
problema grave se manifestará
como fraqueza ou inatividade do
animal
E) Realizar palpação abdominal
de forma negligente e
descuidada
É preciso ter cuidado ao se palpar o abdome; pode
ser fácil causar algum dano, especialmente se o
clínico (1) não tiver conhecimento do problema
médico ou (2) estiver buscando confirmação de
diagnóstico pré-concebido. Por exemplo;
Um cão com esofagite grave pode demonstrar muita
dor e disfagia, mas do mesmo modo outro cão
Yorkshire Terrier hiperativo pode estar sofrendo de
algum desvio portossistêmico e hepatopatia
avançada.
(1) Não ter consciência da presença de tumor
esplênico, que pode se romper durante a palpação
abdominal com consequente hemorragia interna.
(2) Ao realizar a palpação na hipótese de massa
abdominal, considerar a possibilidade de que a
presença de corpo estranho pode ser um diagnóstico
alternativo; se esse corpo estranho for pontiagudo
ou estiver alojado dentro de alguma alça do
intestino, a palpação descuidada pode resultar em
perfuração.
F) Não ser meticuloso na
observação do caso
Em caso em que há histórico de vômito frequente
que desaparece com tratamento sintomático, é
recomendável reservar um tempo para verificar a
presença ou ausência de fezes escuras, o que pode
indicar hemorragia gastrentérica intermitente. A
não-identificação desse sinal pode retardar o
diagnóstico de doença grave, por exemplo, linfoma
gástrico ou gastrenterite linfocítica-plasmocitária
Figura 9. A assim-chamada postura de “oração” é
sugestiva de dor gastroduodenal.
20
Focus_SPECIAL_GI_port:01focuschatp0108FR 12/05/11 15:37 Página 21
A r m a d i l h a s
e m
d i s t ú r b i o s
g a s t r i n t e s t i n a i s
n o
c ã o
descarte
avançada (Figura 8).
Não confundir as fezes com o conteúdo das
glândulas anais. As secreções dos sacos anais
podem ocasionalmente ser negras e permanecer
grudadas à superfície interna do ânus; nesse caso,
essas secreções podem ser confundidas com fezes
contendo sangue digerido. Se necessário, realizar
um exame retal mais profundo com o auxíleo dos
dedos para extrair as fezes de forma apropriada.
Se o proprietário descrever “sangue vivo nas fezes”,
é essencial verificar se o sangue está misturado com
as fezes ou simplesmente as cobre. Se necessário,
efetuar exame seriado das fezes se o sangramento
for intermitente.
Neoplasias gástricas podem ser observadas em
animal com bom estado de pele/pelagem e apenas
perda de peso moderada, enquanto outro animal
com gastrenterite linfocítica-plasmocitária pode se
apresentar em estado particularmente deteriorado,
magro e deprimido e ainda com pelagem em más
condições. Do mesmo modo, um paciente com
pequena neoplasia intestinal pode se apresentar em
um estado geral muito melhor que outro com
enterite inflamatória crônica, em que as alterações
de hipoproteinemia grave e motilidade intestinal
reduzida afetaram acentuadamente a aparência
geral do animal (Figure 10).
G) Colocar o paciente
diretamente na mesa de exame
Conclusão
É imprescindível que o animal seja observado no
chão e sem coleira ou guia antes de colocá-lo na
mesa de exame. Com muita frequência, os cães de
pequeno porte são colocados diretamente na mesa
de exame por seu proprietário; isso pode significar a
perda de sinais valiosos em termos de atitude,
postura e hábitos exploratórios do animal (Figura
9).
Conforme foi afirmado no início deste capítulo, o
exame clínico deve obrigatoriamente fornecer
informações suficientes para permitir que o
veterinário:
•Identifique a gama correta de exames
complementares necessários para possibilitar o
diagnóstico definitivo do problema,
•Atue com a urgência exigida pelo caso,
•Evite a solicitação de exames complementares
desnecessários, que não tenham a chave de solução
para o problema e podem significar um desperdício
de tempo e dinheiro.
H) Assumir que um estado físico
particularmente deteriorado
indica a presença de neoplasia,
enquanto um bom estado físico a
© Davide De Lorenzi
Figura 10. Esse cão Bernese em
bom estado corporal foi
encaminhado por causa de
tenesmo mas, na verdade, ele
estava sofrendo de carcinoma
colônico.
21
Focus_SPECIAL_GI_port:01focuschatp0108FR 12/05/11 15:37 Página 22
A r m a d i l h a s
e m
d i s t ú r b i o s
g a s t r i n t e s t i n a i s
n o
c ã o
> Perguntas a Todd Tams
Quais são as causas mais comuns de esforço para defecar?
Disquezia é definida como defecação dificultosa e/ou dolorosa. Tenesmo refere-se ao esforço persistente ou
prolongado, que costuma ocasionar uma passagem muito pequena de material fecal e, em geral, indica que
o paciente está sofrendo uma sensação de urgência para defecar. Disquezia e tenesmo podem estar
associados com distúrbios alimentares e urogenitais; as causas mais comuns relacionadas ao trato
digestório são doenças do intestino grosso como colite, proctite (inflamação da mucosa retal) e constipação.
Todd Tams
Como os proprietários de pequenos animais frequentemente interpretam o esforço como uma indicação de
constipação, é essencial que o clínico diferencie o esforço inicial de saída por constipação daquele associado
muitas vezes com inflamação do intestino grosso e diarreia. Os clínicos experientes reconhecem que muitos
telefonemas em busca de orientação sobre como tratar um pet constipado em casa, na verdade, envolvem
uma suposição imprecisa feita pelo proprietário. Muito frequentemente, o esforço para defecar é causado
por colite e/ou proctite nesses pacientes. A diferenciação é facilmente feita pela obtenção de histórico
satisfatório e realização de exame físico completo, incluindo palpação abdominal e exame retal.
?
O animal com comprometimento do intestino grosso talvez ofereça outros indícios relevantes. Isso pode
incluir aumento da frequência de defecação, com evacuação de apenas pequenas quantidades de fezes e/ou
material mucoide. Pode haver mudanças de comportamento do animal como resultado da sensação de
urgência para defecar, como um cão normalmente asseado que começa a fazer bagunça na casa enquanto o
proprietário está distante ou indisponível, ou o animal que começa a acordar seu dono com frequência
durante a noite para sair à rua. Um cão pode exibir sinais de irritação perineal, como mordedura na área ou
“scooting”. Qualquer um desses sinais pode ocorrer isoladamente ou em combinação com outros. Colite
aguda é uma causa comum de diarreia em cães e o diagnóstico é facilmente obtido, com base na
apresentação clínica.
22
Focus_SPECIAL_GI_port:01focuschatp0108FR 12/05/11 15:37 Página 23
A r m a d i l h a s
e m
d i s t ú r b i o s
g a s t r i n t e s t i n a i s
n o
c ã o
3. Exames complementares
não instrumentais em
distúrbios gastrintestinais
*
> Resumo
Exceto em casos gastrintestinais não complicados de rápida resolução, o clínico geralmente terá o
desejo de empregar exames complementares para auxiliar no diagnóstico e tratamento da doença
clínica. Há uma grande variedade de exames à disposição para escolher; a seleção dependerá
basicamente das informações obtidas no histórico e no exame clínico (conforme foi abordado nos
capítulos prévios) e, em termos gerais, pode ser classificada como exames laboratoriais, exame
neurológico, exame físico sob sedação e ensaios terapêuticos.
1/ Exames laboratoriais
•Citologia por coloração rápida de Diff Quik
identificará a presença de neutrófilos e outras células
inflamatórias, bem como grandes populações de
certas bactérias em enteropatia aguda
(Campylobacter, Clostrídios formadores de esporos)
ou em episódios de supercrescimento bacteriano do
intestino delgado com doença crônica (Clostrídios). A
presença de fagocitose bacteriana pode ser útil. Em
áreas geográficas endêmicas, talvez se possam
observar amastigotas de Leishmania acompanhados
por células granulomatosas. Os métodos de
coloração ácido-resistente podem identificar
micobactérias e Cryptosporidium.
•Testes antigênicos podem ser diagnósticos para
Giardia se nem os oocistos nem os trofozoítas
estiverem sendo eliminados (os sinais clínicos podem
ou não incluir vômito e/ou diarreia intermitente). Em
cerca de 25% dos casos de Giardia, a análise de
sulfato de zinco com centrifugação será negativa.
Portanto, vale à pena a inclusão de rotina de algum
teste antigênico ou imunofluorescência indireta para
o diagnóstico precoce e preciso de Giardia. Os testes
A) O exame fecal utilizando
amostras facilmente obtidas de
forma não invasiva pode ser
avaliado com rapidez como se
segue:
•Evidência de parasitas; isso pode ser obtido via
exame direto com solução salina (soro fisiológico)
sob microscopia de baixo aumento (para identificar
Trichuris, Ascaris, trofozoítas de Giardia, ou Coccidia),
método de concentração-flotação (para identificar os
ovos desses parasitas) ou, de preferência, flotação
com sulfato de zinco após centrifugação (para ovos
de nematódeos e cistos de Giardia). O sulfato de
zinco com centrifugação é, sem dúvida, superior à
simples flotação por gravidade para o diagnóstico de
parasitas intestinais, sendo mais bem realizado por
técnicos laboratoriais experientes. O volume
adequado de amostra para o exame de fezes deve ser
de, no mínimo, 2-5 gramas (Figuras 1 e 2).
23
Focus_SPECIAL_GI_port:01focuschatp0108FR 12/05/11 15:37 Página 24
e m
d i s t ú r b i o s
g a s t r i n t e s t i n a i s
n o
c ã o
© University of Iowa
© University of Pennsylvania
A r m a d i l h a s
Figura 1. Giardia.
Figura 2. Coccídia (Isospora canis).
antigênicos também são de grande utilidade para o
diagnóstico de parvovírus (que se manifesta sob a
forma de vômito e diarreia hemorrágicos profusos
agudos em animais jovens ou imunossuprimidos).
Notar que, mais frequentemente que não, o
hemograma completo está dentro dos limites de
normalidade.
Análise eletrolítica:
•Isso pode identificar o seguinte: hipocalemia (p. ex.,
gastrenterite secretória aguda, doença renal crônica)
ou hipercalemia e hiponatremia (vômito e/ou diarreia
em hipoadrenocorticismo).
B) A coleta de amostra de sangue
para hemograma, análise
eletrolítica e bioquímica sérica
deve ser considerada se os
indícios do exame clínico
indicarem a utilidade desses
exames:
Perfil bioquímico: os itens expostos a seguir
são dignos de nota em casos de distúrbios
gastrintestinais:
•Proteínas totais baixas com albumina
moderadamente baixa (doença inflamatória intestinal
com má-absorção ou má digestão) ou albumina muito
baixa (enteropatia com perda proteica, linfoma
intestinal); proteínas totais normais, mas albumina
baixa (disfunção renal ou hepática atribuída à doença
crônica);
ou
hipoproteinemias
absolutas
(hemorragia).
•Altos níveis de BUN (de Blood Urea Nitrogen, sigla
conhecida em inglês, que significa nitrogênio ureico
sanguíneo ou ureia) e creatinina (vômito com ou sem
diarreia em doença renal) ou BUN elevado (digestão
de sangue em hemorragia gastrentérica grave).
•Elevação moderada das enzimas hepáticas
(ocasionalmente em doença inflamatória intestinal
crônica (Bush, 1997), deficiência hepática como em
desvios portossistêmicos, pancreatite), ou aumento
muito acentuado dessas enzimas (hepatopatia
Hemograma completo:
•A realização de eritrograma pode ajudar a
identificar e pesquisar o seguinte: anemia por perda
sanguínea (doença ulcerativa gástrica ou intestinal),
anemia por doença crônica (má-absorção de
nutrientes),
anemia
moderada
(hipoadrenocorticismo), ou hematócrito elevado (p.
ex., em virtude de gastrenterite aguda com
desidratação). A obtenção de leucograma (ou seja,
contagem de leucócitos) pode extrair as seguintes
informações: leucocitose (p. ex., gastrenterite
bacteriana
aguda,
como
salmonelose,
hipoadrenocorticismo, doença inflamatória intestinal
crônica), eosinofilia (enterite eosinofílica,
endoparasitismo,
hipoadrenocorticismo)
ou
linfopenia absoluta (em alguns casos de
linfangiectasia).
24
Focus_SPECIAL_GI_port:01focuschatp0108FR 12/05/11 15:37 Página 25
A r m a d i l h a s
e m
d i s t ú r b i o s
g a s t r i n t e s t i n a i s
n o
c ã o
•Os ensaios séricos de cobalamina e folato caninos
podem ser de grande utilidade, mas talvez sejam
superinterpretados. Baixos níveis de cobalamina são
observados em casos de insuficiência pancreática
exócrina, supercrescimento bacteriano na porção
proximal do intestino delgado ou enteropatia na
porção distal desse intestino. Altos níveis de folato
ocorrem no supercrescimento bacteriano na porção
proximal do intestino delgado, enquanto níveis mais
baixos que a faixa de normalidade talvez sejam vistos
em enteropatia na porção proximal desse intestino.
•Valores séricos pré e pós-prandiais de ácidos
biliares são úteis para avaliar a função hepática na
suspeita de deficiência hepática.
primária). Os animais hepatopatas também podem
apresentar níveis enzimáticos normais ou apenas
levemente aumentados.
•Hiperglicemia (diabetes mellitus) ou hipoglicemia
(que talvez necessite de amostragem seriada para
detectar; p. ex., diarreia em septicemia).
C) A urinálise também deve ser
realizada, de preferência, ao
mesmo tempo da amostragem de
sangue, para avaliar a densidade
e as proteínas urinárias:
•Densidade normal (presente em azotemia renal) ou
densidade baixa (doença renal).
•Proteinúria alta acompanhada por hipoproteinemia
(síndrome nefrótica), proteinúria moderada (doença
sistêmica) ou ausência de proteinúria (doença
inflamatória intestinal crônica, linfangiectasia,
linfoma intestinal).
Exames bioquímicos mais específicos podem ser
realizados se houver a possibilidade de certos
distúrbios orgânicos ou sistêmicos ao exame clínico
e/ou nos exames complementares preliminares e
caso seja desejável descartar ou confirmar essas
suspeitas. Esses exames podem incluir o seguinte:
Em geral, os exames de hemograma, perfil
bioquímico e urinálise devem ser solicitados se
houver sinais de doença sistêmica, como
poliuria/polidipsia, perda de apetite, perda de peso,
vômito e/ou diarreia profusa. Mesmo se os sinais
clínicos forem brandos e/ou apenas intermitentes,
esses exames de triagem sempre são valiosos para
proporcionar uma avaliação mais completa do estado
de saúde geral do paciente.
•Teste de estimulação com ACTH em cães com
diarreia
e
vômito,
sugestivos
de
hipoadrenocorticismo.
•Teste do Tensilon® para cães com regurgitação ou
disfagia, e/ou fraqueza muscular, em que há suspeita
de miastenia grave.
•Anticorpos antiacetilcolina (disponíveis atualmente
apenas em laboratórios norte-americanos). Notar que
a regurgitação por
D) Provas de função
gastrintestinal:
dilatação do esôfago ou a disfagia por disfunção da
laringe pode ser o único sintoma clínico observado no
início de miastenia grave adquirida (Shelton, 2002).
•Testes de anticorpos antinucleares (AAN) para cães
com artralgia (dor articular) ou na suspeita de
poliartrite imunomediada (que pode se manifestar
sob a forma de disfagia ou regurgitação).
•O ensaio de imunorreatividade semelhante à da
tripsina (TLI) é útil como prova de função pancreática.
Em alguns casos de pancreatite, essa
imunorreatividade estará muito alta, mas será
constantemente mais baixa que o normal em casos
de insuficiência pancreática exócrina.
•O ensaio de imunorreatividade da lipase
pancreática (PLI), recém-desenvolvido, é mais preciso
para o diagnóstico de pancreatite, embora se deva
notar que esse exame esteja disponível atualmente
apenas em laboratórios norte-americanos
(www.cvm.tamu.edu/gilab).
2/ Exames neurológicos
Pode haver a necessidade de exame neurológico em
casos de:
•Suspeita de lesões focais da medula oblonga, o que
25
Focus_SPECIAL_GI_port:01focuschatp0108FR 12/05/11 15:37 Página 26
A r m a d i l h a s
e m
d i s t ú r b i o s
Figura 3.
Golden
Retriever sob
sedação para
exame da
cavidade oral.
g a s t r i n t e s t i n a i s
n o
c ã o
A resposta a ensaio terapêutico pode ser valiosa em
alguns casos, tais como:
•Na suspeita de parasitose “oculta” (utilizando
fembendazol ou febantel para tratamento de Giardia,
Trichuris ou Ascaris).
•Na suspeita de hipersensibilidades alimentares
(utilizando dietas hidrolisadas ou dietas de
eliminação) ou intolerâncias alimentares (usando
dietas isentas de lactose ou de glúten).
Notar que a administração de antibióticos não deve
ser considerada como ensaio terapêutico.
5/ Armadilhas frequentes
em exames
complementares
© Valérie Freiche
pode
se
manifestar sob a forma de disfagia, acompanhada
por ataxia, fraqueza dos membros e
A) Protelar o momento oportuno
para o início dos exames
complementares
déficits proprioceptivos.
•Suspeita de lesões dos nervos cranianos; talvez
haja necessidade de avaliação desses nervos nos
casos em que se observam distúrbios de deglutição,
disfagia/regurgitação e problemas com a língua;
particularmente, é recomendável a avaliação dos
nervos glossofaríngeo (IX), vago (X) e hipoglosso (XII).
•Movimentos anormais durante a deambulação; isso
pode ser atribuído à fraqueza muscular, que talvez se
manifeste como disfagia.
As informações obtidas a partir do histórico e do
exame clínico devem indicar a urgência com que
esses exames devem ser realizados. O diagnóstico
precoce pode possibilitar o fornecimento de
prognóstico positivo e permitir a obtenção de
resposta satisfatória ao tratamento. Se o animal
estiver em má condição física ou se houver sintomas
de vômito ou diarreia por 1 ou 2 semanas ou se o
paciente revelar sinais de dor abdominal, anemia ou
choque, é óbvio que esses exames devem ser
rodados imediatamente. Contudo, o momento de
investigação do problema por meio de exames
também pode depender do nível de preocupação
demonstrado pelo proprietário.
3/ Exame físico sob
sedação
Isso pode ser necessário para examinar:
•Cavidade oral, faringe, laringe ou criptas tonsilares
(em casos de disfagia) (Figura 3).
•O abdome, especialmente para permitir a palpação
abdominal mais eficiente em animal tenso ou acima
do peso ideal.
•Região anorretal quando o exame se mostra
doloroso (nesse caso, os sinais podem incluir
disquezia ou hematoquezia).
B) Falhar em fazer a triagem
completa de parasitas GI,
utilizando os exames fecais mais
sensíveis
O clínico pode simplesmente aceitar um único
resultado negativo e descartar a possibilidade de
parasitas. É essencial que a presença de parasitas
intestinais seja identificada o mais cedo possível e,
se necessário, é recomendável a repetição dos
exames em busca de parasitas.
4/ Ensaio terapêutico
26
Focus_SPECIAL_GI_port:01focuschatp0108FR 12/05/11 15:37 Página 27
A r m a d i l h a s
e m
d i s t ú r b i o s
g a s t r i n t e s t i n a i s
n o
c ã o
> Perguntas a Todd Tams
Qual é o erro mais comum cometido no diagnóstico de Giardia?
Muitos casos de Giardia em cães são erroneamente diagnosticados por causa do emprego de exames diagnósticos fecais
insensíveis. O exame de fezes (padrão) composto de flotação por gravidade, sem centrifugação, não é um teste muito
sensível para o diagnóstico de Giardia, e algumas infecções por nematódeos também podem passar despercebidas,
especialmente na presença de baixas contagens de ovos. Os estudos demonstraram que as contagens de ovos obtidas com
as técnicas de centrifugação são 2,4 a 6 vezes mais altas que aquelas adquiridas com método de flotação por sedimentação.
Todd Tams
O melhor método de triagem de uma única amostra fecal em busca de qualquer tipo de parasita intestinal em cão com
diarreia é realizar:
1. Esfregaço direto em salina (examinar a amostra fecal fresca dentro de 1 hora após a evacuação)
2. Ensaio de sulfato de zinco com centrifugação
3. Teste antigênico de Giardia
?
Notas:
1. Apesar de não ser um teste diagnóstico altamente sensível, um único esfregaço direto é um método simples de realizar e,
caso se observe a presença de trofozoítas móveis, torna-se possível o diagnóstico rápido. Notar que os trofozoítas de Giardia
precisam ser diferenciados de trichomonas.
2. O sulfato de zinco com centrifugação é bem mais sensível para o encontro de cistos de Giardia, em comparação ao teste
de simples flotação por gravidade. Vários estudos demonstraram que, em um único teste de centrifugação, o rendimento
diagnóstico em cão com infecção por Giardia será em torno de 75%. Se o ensaio de centrifugação for repetido diariamente
por 3 dias, o rendimento aumentará para cerca de 95%.
3. O teste ELISA para detecção do antígeno da Giardia é um exame excelente para pesquisa desse parasita. Quando
combinado com ensaio de centrifugação, o rendimento diagnóstico tanto para Giardia como para nematódeos é muito alto.
Portanto, o método mais eficiente — e basicamente econômico — para fazer a triagem precisa de uma ampla gama de
parasitas intestinais, inclusive Giardia, em animais consiste na realização de todos os três exames listados acima. Para
triagem anual e semestral de rotina em animais sem quaisquer sinais clínicos anormais, o ensaio de sulfato de zinco com
centrifugação constitui o teste básico de escolha. Não efetuamos esfregaços diretos em salina com rotina em amostras
fecais não diarreicas.
C) Não avaliar as fezes quando o
vômito é o único sinal clínico
D) Considerar que o perfil
bioquímico esteja “normal” sem
avaliar as proteínas
Muitos distúrbios entéricos ou colônicos,
independentemente de terem origem parasitária,
infecciosa ou inflamatória, podem se apresentar com
vômito como o sinal mais precoce. No caso de
parasitose por Giardia, os sintomas podem consistir,
sobretudo, em vômito, com fezes amolecidas
intermitentes. A importância da avaliação fecal, no
entanto, não pode ser superenfatizada.
A obtenção de valor normal para as proteínas totais
(PT) no perfil bioquímico não deve conferir, por si só, a
submissão total ao resultado. Pode haver
hipoalbuminemia
atribuída
a
problemas
gastrintestinais sem hipoglobulinemia concomitante.
Em qualquer caso clínico, sempre devem ser
avaliados os níveis de proteínas totais, albumina e
27
Focus_SPECIAL_GI_port:01focuschatp0108FR 12/05/11 15:37 Página 28
A r m a d i l h a s
e m
d i s t ú r b i o s
g a s t r i n t e s t i n a i s
n o
c ã o
de resultado “normal”
globulinas em perfil bioquímico. Isso fará com que o
clínico diferencie entre hipoalbuminemia
compensatória (p. ex. em casos de Leishmaniose ou
Erliquiose) ou hipoproteinemia absoluta, em que há
redução tanto da albumina como da globulina.
Alternativamente, se a hipoalbuminemia for
detectada, é recomendável a avaliação dessa
anormalidade junto com a urinálise para diferenciar
síndrome nefrótica, ou doença hepática, de possível
enteropatia com perda proteica. O último distúrbio
pode se agravar drasticamente em questão de dias
se o diagnóstico e o tratamento apropriados não
forem iniciados quando a concentração de albumina
estiver abaixo de 20 g/L (2 g/dL).
Em cães, valores abaixo de 2 mg/L são
diagnosticados com insuficiência pancreática
exócrina. Valores entre 3,5 e 5 mg/L podem refletir
doença pancreática subclínica, como pancreatite
crônica. Esses pacientes podem, mais tarde, evoluir
para insuficiência pancreática exócrina quando os
valores de TLI estiverem entre 2 e 3,5 mg/L. É
essencial enfatizar que, se os resultados forem
ambíguos, o teste de TLI deverá ser repetido 1 mês
depois.
H) Efetuar o ensaio de TLI para o
diagnóstico de casos suspeitos de
pancreatite depois de alguns dias
de tratamento
E) Não realizar o exame de TLI
para suspeita de insuficiência
pancreática exócrina em todos os
casos com suspeita de doença
crônica do intestino delgado
O ensaio de TLI deve ser obrigatoriamente realizado
em amostra de sangue coletada no início da doença,
pois os altos níveis de tripsina desaparecem com
rapidez assim que a terapia intravenosa for instituída.
Em casos de possível diagnóstico de pancreatite, é
recomendável a instituição de terapia apropriada o
mais rápido possível em vez de adiá-la até que se
obtenha o resultado. O teste de PLI (disponível pela
Universidade A & M do Texas) é atualmente o teste
mais sensível disponível para o diagnóstico de
pancreatite.
A omissão desse exame simples pode induzir ao
diagnóstico errado, à falha terapêutica e, nos casos
mais graves, à deterioração da mucosa intestinal.
Pode ser muito fácil confundir insuficiência
pancreática exócrina com doença inflamatória
intestinal. Omitir o ensaio de imunorreatividade
semelhante à da tripsina também pode significar que
o animal fique sujeito a ensaios terapêuticos
repetidos com enzimas pancreáticas sem resposta, o
que pode levar ao abandono do tratamento pelo
proprietário ou desespero do clínico.
I) Superestimar os resultados de
cobalamina e folato no
diagnóstico de doença
inflamatória intestinal
F) Avaliar o exame de TLI para o
diagnóstico de insuficiência
pancreática exócrina sem
observar o período de jejum
necessário antes da amostragem
Pode ser muito fácil interpretar erroneamente os
resultados dos testes de cobalamina e/ou folato. Por
exemplo, valores de cobalamina abaixo da faixa de
normalidade podem ser atribuídos à insuficiência
pancreática exócrina e não ao supercrescimento
bacteriano do intestino delgado. Nem todos os cães
com enteropatia exibem resultados anormais de
cobalamina e folato. Nem todas as enteropatias, que
provocam má-absorção de vitaminas, são
suficientemente graves ou prolongadas a ponto de
reduzir os depósitos após a concentração dessas
A realização do teste de TLI após jejum de apenas 6
horas pode gerar resultados ambíguos em muitos
casos; é recomendável o jejum mínimo de 10 horas
antes da amostragem.
G) Descartar insuficiência
pancreática exócrina na obtenção
28
Focus_SPECIAL_GI_port:01focuschatp0108FR 12/05/11 15:37 Página 29
A r m a d i l h a s
e m
d i s t ú r b i o s
g a s t r i n t e s t i n a i s
n o
c ã o
investigação do problema se o ensaio não resultar em
resposta satisfatória. Sem planejamento futuro, pode
ser simplesmente tentador prescrever um segundo
remédio ou dieta, outra vez sem sucesso. Isso
desperdiçará um tempo diagnóstico valioso e pode
fazer com que o proprietário perca a confiança no
veterinário se outros exames complementares forem
oferecidos mais tarde, independentemente se eles
forem laboratoriais ou instrumentais.
vitaminas no corpo (Melgarejo, 2002).
J) Realizar o tratamento com base
nos resultados de coprocultura
bacteriana ou no conteúdo
duodenal
A técnica para coleta de amostras não é simples, pois
requer endoscopia ou intervenção cirúrgica,
particularmente na obtenção de amostras direto do
intestino delgado. Se a amostragem não for feita com
rigor e método estéril, os resultados das culturas
podem vir a ser de pouca utilidade na seleção de
tratamento apropriado.
N) Não ter conhecimento das
diferenças nos valores
laboratoriais de referência entre
cães e gatos
K) Descartar hepatopatia em caso
com vômito hemorrágico ou
melena quando as enzimas
hepáticas estão normais ou
apenas levemente elevadas
É importante ter em mente as diferenças entre cães e
gatos ao se interpretar os resultados das amostras de
sangue; por exemplo, os baixos níveis de proteínas
totais observados em cães com doença inflamatória
intestinal não são vistos em gatos acometidos pela
mesma doença. O clínico também deve estar
familiarizado com os parâmetros normais do
laboratório escolhido.
Talvez haja necessidade da análise de ácidos biliares
caso esteja buscando a identificação da doença
hepática responsável por hemorragia gástrica e
perda das camadas protetoras da mucosa gástrica;
parâmetros bioquímicos isolados podem não ser
suficientes para excluir envolvimento hepático.
Conclusão
Um clínico astuto escolherá sabiamente a partir da
ampla seleção de exames complementares
disponíveis para assegurar que a realização dos
testes mais úteis, sem atraso, auxilie no diagnóstico
preciso de doença gastrintestinal. Também é preciso
ter cuidado ao se interpretar os resultados para
garantir que as informações obtidas a partir dos
exames sejam utilizadas de forma correta.
L) Selecionar os exames
laboratoriais de acordo com a
raça
A “visão em túnel” pode induzir à escolha seletiva de
exames laboratoriais pelo clínico, com base na raça
do paciente, p. ex., realizar ensaios de TLI apenas em
cães da raça Pastor Alemão, análises de ácidos
biliares somente em Yorkshire Terrier ou Schnauzer
Miniatura ou níveis de albumina exclusivamente em
Sharpei.
M) Não planejar a próxima etapa
após ensaio terapêutico
Caso se efetue algum ensaio terapêutico (seja
farmacológico ou nutricional), o clínico deve estar
preparado para a próxima etapa requerida na
29
Focus_SPECIAL_GI_port:01focuschatp0108FR 12/05/11 15:37 Página 30
A r m a d i l h a s
e m
d i s t ú r b i o s
30
g a s t r i n t e s t i n a i s
n o
c ã o
Focus_SPECIAL_GI_port:01focuschatp0108FR 12/05/11 15:37 Página 31
A r m a d i l h a s
e m
d i s t ú r b i o s
g a s t r i n t e s t i n a i s
n o
c ã o
4. Técnicas de diagnóstico
por imagem e endoscopia do
sistema digestório: instruções
de uso
> Resumo
*
Vômito e diarreia são sintomas inespecíficos frequentemente observados. Esses sintomas nem
sempre são causados por doenças que se originam primariamente no trato gastrintestinal e,
além disso, o mesmo sintoma pode ser causado por doenças que envolvem partes diferentes do
trato gastrintestinal.
Assim que o paciente for submetido a exame físico completo e avaliação extensiva dos
parâmetros hematológicos, bioquímicos e sorológicos, as técnicas diagnósticas instrumentais
possibilitam uma investigação mais detalhada e frequentemente conclusiva da doença.
Há muitas técnicas distintas de diagnóstico por imagem, disponíveis para uso na identificação
de uma ampla variedade de doenças, capazes de afetar o trato gastrintestinal, como:
radiografia, ultrassonografia, endoscopia e tomografia computadorizada (TC), das quais cada
uma delas está associada com indicações específicas.
Introdução
A finalidade deste capítulo é ilustrar as indicações,
bem como os prós e contras das várias técnicas de
diagnóstico por imagem, enfatizando a importância do
uso correto dessas técnicas e salientando a natureza
complementar das informações fornecidas por elas.
1/ Exame radiográfico
A radiografia abdominal é indicada sempre que os
sinais clínicos ou os resultados laboratoriais sugerirem
a presença de doença com envolvimento de estrutura
ou órgão situado dentro do abdome. Mais
especificamente, os sinais clínicos sugestivos de
31
doença gastrintestinal são regurgitação, vômito,
diarreia, tenesmo e anorexia ou perda de peso, em que
a causa não foi identificada com base nos exames
laboratoriais mais precoces. Além disso, deve-se
lembrar que outros sinais clínicos inespecíficos, como
massas abdominais palpáveis, organomegalia,
distensão abdominal, dor à palpação abdominal,
anemia, febre de origem desconhecida e
poliuria/polidipsia, podem ser causados por doenças
que afetam primariamente o sistema gastrintestinal.
Como todos os órgãos situados no abdome
apresentam a radiopacidade típica de tecido mole ou
líquido, a interpretação de radiografias abdominais é
frequentemente difícil e inconclusiva, devido à
impossibilidade de distinguir um órgão do outro com
segurança. Alterações patológicas tornam-se
apreciáveis ao exame radiográfico apenas se a doença
causar alteração suficiente em termos de densidade,
tamanho, formato ou posição do órgão envolvido, de
Focus_SPECIAL_GI_port:01focuschatp0108FR 12/05/11 15:37 Página 32
e m
d i s t ú r b i o s
n o
c ã o
após a administração do bário e às consideráveis
variações entre os cães na velocidade de
esvaziamento gástrico e do trânsito gastrintestinal
(Konde, 2003).
O desempenho correto dessa técnica é uma questão
complexa (ver Quadro 1) e, muitas vezes, requer
sedação do paciente, particularmente ao lidar com um
cão nervoso. É importante notar que, se o estudo não
for bem realizado, a interpretação das radiografias
pode não ser proveitosa — e, possivelmente,
equivocada — e não diagnóstica. Além disso, a
sedação por si só é capaz de alterar o tempo de
trânsito GI. Por essas razões, os estudos com contraste
estão cada vez mais sendo substituídos por outras
técnicas de investigação, particularmente a
ultrassonografia. Uma alternativa relativamente
a.
b.
© Davide De Lorenzi
© Davide De Lorenzi
modo que ele possa ser claramente diferenciado das
estruturas circunjacentes.
Alguns órgãos, no entanto, podem ter conteúdo não
líquido, que facilita a distinção entre eles e as
estruturas circunjacentes. Isso ocorre tipicamente na
presença de gases no estômago e nas alças intestinais
ou fezes no cólon e reto. A administração de meios de
contraste por via oral ou retal é uma técnica
frequentemente utilizada para a identificação de
alterações ou anormalidades que não podem ser
avaliadas por meio de radiografia simples. Notar,
entretanto, que os estudos com contraste devem ser
efetuados somente depois das radiografias simples. O
exame do trato digestório superior com meio de
contraste costuma ser realizado após a administração
de bário sob a forma líquida, mas as principais
indicações para esse tipo de estudo são quando o
clínico deseja procurar por alterações anatômicas,
evidências de corpos estranhos radiopacos (Figura 1),
obstruções parciais (Figura 2), ou tempos de trânsito
intestinal anormais. Os exames do trato digestório
inferior (cólon e reto) com meio de contraste são
executados em uma frequência muito menor, em parte
por serem raramente indicados, mas sobretudo, por
causa do emprego mais fácil e mais proveitoso de
técnicas diagnósticas endoscópicas.
Os principais problemas para interpretar corretamente
as radiografias envolvendo os meios de contraste são
secundários à falha no preparo correto do paciente, à
administração de quantidade insuficiente do meio de
contraste, à obtenção de pouquíssimas radiografias
g a s t r i n t e s t i n a i s
© Davide De Lorenzi
A r m a d i l h a s
Figura 1. Corpo estranho gástrico
radiopaco (isca de peixe).
Figura 2a. Radiografia pós-contraste; o
bário acumula-se em um segmento
extremamente dilatado do íleo por
oclusão intestinal.
Figura 2b. Mesmo caso da figura 2a;
animal em cirurgia; hérnia inguinal
crônica com alça ileal encarcerada.
32
Focus_SPECIAL_GI_port:01focuschatp0108FR 12/05/11 15:37 Página 33
A r m a d i l h a s
e m
d i s t ú r b i o s
g a s t r i n t e s t i n a i s
n o
c ã o
Quadro 1. Técnica para realização de estudos do trato digestório superior com meios de contraste
•
• Jejum de, no mínimo, 12 horas
• Sedação, quando necessária, dependendo do temperamento do paciente
• Administração de bário sob a forma líquida (solução a 25 ou 40% pronta para usar) por meio de tubo
orogástrico ou nasogástrico, na dose de 5-10 mL/kg de peso
• Checar (por meio de palpação ou radiografia) se o tubo está posicionado corretamente antes de
administrar o bário
• Obter as radiografias imediatamente após a administração, depois em 15 minutos, 30 minutos, 1
hora, 2 horas, 3 horas e, em seguida, periodicamente, de hora em hora, até que o contraste atinja o
cólon (isso normalmente leva de 90 a 270 minutos (Miyabayashi, 1984) embora haja relatos de
tempos de esvaziamento gástrico variados de 5 a 10 horas em pacientes saudáveis (Konde, 2003),
(Arnjeberg, 1992). Obter projeções laterais e ventrodorsais (VD) em cada período de tempo.
separar o corpo da vértebra em duas partes simétricas.
Por fim, é bom lembrar que existem órgãos do trato
gastrintestinal não situados no abdome, tais como a
faringe e o esôfago. Portanto, uma avaliação completa
do trato gastrintestinal também deve incluir
radiografias do pescoço e do tórax (Figura 3).
Em suma, a investigação radiográfica do trato
gastrintestinal deve ser considerada como um sistema
de triagem rápido e econômico. No entanto, em virtude
das inúmeras limitações dessa técnica, a radiografia
simples raramente possibilita a obtenção de
diagnóstico definitivo (conclusivo) e, por essa razão, há
necessidade de recorrer a outras modalidades
diagnósticas mais complexas.
© Davide De Lorenzi
recente ao uso de bário sob a forma líquida para
estudo da motilidade gastrintestinal consiste em
esferas de polietileno impregnadas de bário (EPIB)
(Robertson, 2000). Essas esferas são pequenas bolas,
que variam de diâmetro e são inseridas em cápsulas
gelatinosas misturadas com a ração do animal. Essa
técnica de contraste é utilizada não só para identificar
a presença de estenose parcial do trato gastrintestinal,
mas também para avaliar o esvaziamento gástrico e a
velocidade dos tempos de trânsito orocolônico
(Robertson, 2000), embora sua popularidade varie
entre os clínicos — essa técnica não é a preferência
pessoal do autor.
As radiografias abdominais para estudar o trato
gastrintestinal sempre devem ser obtidas com, no
mínimo, duas projeções, geralmente ventrodorsal e
lateral (com o animal em decúbito lateral direito) e no
final da expiração. É de fundamental importância o
conhecimento da anatomia radiográfica normal e da
possível ocorrência de variações morfológicas (muitas
vezes, consideráveis e enganosas), dependendo da
posição do animal. Antes de interpretar alguma
radiografia em busca de quaisquer anormalidades ou
alterações, é essencial avaliá-la quanto à exposição e
ao posicionamento satisfatórios. Em particular, o
posicionamento correto da projeção lateral é avaliado,
verificando-se se os processos transversos das
vértebras lombares e as asas do íleo estão sobrepostos
entre si, enquanto o posicionamento correto da
projeção ventrodorsal é estimado, examinando-se o
processo espinhoso de cada vértebra, o que deve
Figura 3. Grande corpo estranho
esofágico (osso).
33
Focus_SPECIAL_GI_port:01focuschatp0108FR 12/05/11 15:37 Página 34
A r m a d i l h a s
e m
d i s t ú r b i o s
2/ Exame ultrassonográfico
g a s t r i n t e s t i n a i s
n o
c ã o
•a ausência de riscos biológicos tanto para o operador
como para o paciente
•a possibilidade de realização do exame sem recorrer
à anestesia ou sedação
•o potencial de check-ups seriados sob custos
acessíveis
•o potencial de avaliação da atividade peristáltica do
estômago e intestino, exame da estratificação das
paredes gástrica e intestinal, bem como avaliação de
todas as outras estruturas abdominais
(Figura 4)
•a possibilidade de obtenção de amostras de
excelente qualidade para exame citológico e biopsia
com técnica minimamente invasiva
Entre as principais limitações dessa técnica de
diagnóstico por imagem ao se avaliar o trato
gastrintestinal, destacam-se:
•a dificuldade de observação de todas as áreas do
estômago e dos intestinos
•a ausência de sinais ultrassonográficos
patognomônicos para muitas doenças possíveis
•a dificuldade de localização exata das lesões do trato
intestinal pela falta de pontos de referência
•a presença de gases como possível fator limitante na
visualização das vísceras pela possibilidade de
formação de artefatos, que impedem a avaliação
satisfatória das paredes intestinais distais à sonda
•a dificuldade de identificação e avaliação do
pâncreas
Os exames ultrassonográficos do sistema digestório
devem ser obrigatoriamente realizados por meio de
instrumentos adaptados com sondas de alta resolução
e alta frequência (5-7,5 MHz), já que esse tipo de
instrumento possibilita a visualização ideal das
paredes gastrintestinais e pode permitir a
identificação de alterações ou anormalidades até
mesmo muito pequenas.
Se o exame ultrassonográfico for planejado com
antecedência, é recomendável que o paciente fique
em jejum por, no mínimo, 24 horas para reduzir a
quantidade de gás e a presença de material fecal.
Como já foi mencionada, a presença de gases é um
dos principais fatores limitantes na ultrassonografia do
estômago e intestino em virtude da formação de
artefatos que, muitas vezes, impedem a visualização
de toda a extensão do trato digestório.
© Davide De Lorenzi
Até pouco tempo, a ultrassonografia era considerada
uma má escolha para avaliar o trato gastrintestinal:
por muito tempo, a presença de gases, alimentos e
fezes, juntamente com a baixa definição obtida pelos
primeiros equipamentos ultrassonográficos, formava
um obstáculo para permitir a avaliação precisa e
completa de todo o trato gastrintestinal. Contudo, o
progresso tecnológico, aliado com o aumento da
experiência por parte dos ultrassonografistas, fez com
que a qualidade das imagens adquiridas melhorasse
muito e, no momento, essa técnica de investigação é
uma das mais frequentemente utilizadas para avaliar
doenças do trato gastrintestinal.
Um vasto número de publicações na literatura
veterinária especializada revela que a ultrassonografia
pode ser utilizada de forma vantajosa para avaliar
doenças com envolvimento do estômago e intestino
(Penninck, 1990 e 1995; Hudson, 1995). Problemas
como neoplasias, corpos estranhos, intussuscepção
ileal, doenças inflamatórias e anormalidades
congênitas podem ser definidos com maior clareza —
e, muitas vezes, diagnosticados de forma definitiva —
graças a esse método.
Em comparação com outras técnicas de diagnóstico
por imagem, a ultrassonografia tem muitas vantagens,
incluindo:
Figura 4. Durante o exame
ultrassonográfico, todas as estruturas intraabdominais podem ser visualizadas: nessa
imagem, é evidente a presença de
linfonodo mesentérico infartado.
34
Focus_SPECIAL_GI_port:01focuschatp0108FR 12/05/11 15:37 Página 35
A r m a d i l h a s
e m
d i s t ú r b i o s
a.
n o
c ã o
desde o estômago até o ânus, avaliando-o tanto em
termos de anatomia normal como em relação a
alterações patológicas.
O exame ultrassonográfico do sistema gastrintestinal
sempre deve considerar a estratificação e a espessura
da mucosa, o conteúdo do lúmen, o peristaltismo e os
linfonodos regionais.
A estratificação do trato gastrintestinal é uma
constante, correspondendo a várias camadas distintas
identificáveis ao exame histológico: deslocando-se do
lúmen para o lado de fora, é possível identificar uma
primeira linha hipoecogênica, que equivale à área de
transição entre o lúmen e a mucosa intestinal, uma
segunda linha hipoecogênica, que corresponde à
mucosa, uma terceira linha hiperecogênica, que se
trata da submucosa, uma quarta linha hipoecogênica,
que se refere à camada muscular e uma quinta e
última linha hiperecogênica, que diz respeito às
camadas serosa e subserosa.
É preciso enfatizar, entretanto, que a identificação
correta de todas as camadas listadas anteriormente
depende de vários fatores, incluindo a presença de
gases e o conteúdo do lúmen. A impossibilidade de
identificação de todas as estratificações não significa
necessariamente a existência de distúrbio patológico
(Figuras 5).
A espessura da mucosa varia, dependendo de qual
parte do trato gastrintestinal está sendo examinada.
No estômago, essa espessura varia de 3 a 5 mm, no
intestino delgado, de 2 a 3 mm e no intestino grosso,
de 2 a 3 mm (Figura 6).
Os movimentos peristálticos podem ser realçados com
© Davide De Lorenzi
© Davide De Lorenzi
Uma dica para melhorar a definição desses órgãos é
mudar a posição do animal e posicionar a sonda sobre
o lado pendente do paciente; isso é obtido de forma
mais eficiente, lançando mão de mesa de
ultrassonografia com um buraco (abertura) nela. Nesse
caso, o gás tende a subir para a face dorsal do animal,
reduzindo com isso os artefatos, enquanto o trato
gastrintestinal tende a repousar contra a parede
abdominal pendente.
Outro método útil para deslocar o gás situado nas
alças intestinais que ficam em contato com a
superfície corporal consiste na compressão gradativa e
delicada da área a ser examinada, o que deslocará o ar,
mas não as alças intestinais a serem examinadas.
Essa técnica pode, no entanto, causar certa
quantidade de dor, tornando difícil, se não impossível,
o uso em pacientes com abdome sensível.
Foram descritas técnicas mais complexas para realçar
as paredes gástricas; por exemplo, pela remoção do ar
com pequena sonda orogástrica e preenchimento do
órgão com água (Penninck, 1995) ou pela
administração de bário (meio de contraste) antes da
avaliação ultrassonográfica (Hudson, 1995) para
melhorar a qualidade do exame. Tais métodos, todavia,
raramente são utilizados na prática clínica, em parte
por exigirem sedação frequente do animal.
A varredura metódica e padronizada do abdome é um
pré-requisito essencial para a avaliação adequada do
trato gastrintestinal: todos os quadrantes devem ser
avaliados por meio de varreduras longitudinais,
transversas e oblíquas do paciente. Por esse método,
costuma ser possível acompanhar o trato digestório,
g a s t r i n t e s t i n a i s
Figuras 5a e 5b. Linfoma intestinal: a mucosa está espessada e a falta da
estratificação típica pode ser visualizada.
35
b.
Focus_SPECIAL_GI_port:01focuschatp0108FR 12/05/11 15:37 Página 36
A r m a d i l h a s
e m
d i s t ú r b i o s
g a s t r i n t e s t i n a i s
n o
c ã o
Figura 6. O aumento moderado de
espessura da mucosa duodenal está
associado com estratificação normal
nesse caso de enterite
linfoplasmocitária.
© Davide De Lorenzi
por imagem. A ultrassonografia, portanto, deve ser
realizada em combinação com radiografias de boa
qualidade e avaliação endoscópica, mas
frequentemente fornecerá informações peculiares
complementares àquelas obtidas pelas outras técnicas
mencionadas.
clareza por meio ultrassonográfico. Quatro a cinco
ondas peristálticas por minuto são consideradas
normais para o estômago e o duodeno, mas 1 a 3
contrações para o restante do intestino delgado,
enquanto o intestino grosso não costuma exibir
qualquer atividade peristáltica evidente.
Em relação ao conteúdo intestinal, podem ser
identificados três tipos diferentes de imagem
ultrassonográfica: gás, muco e líquido. Embora a
presença de líquido e muco facilite a melhor
interpretação das paredes abdominais, a presença de
gás forma uma interface altamente hiperecogênica
com a mucosa e pode produzir o que é conhecido como
artefatos de “anel para baixo”.
Como já foi mencionado previamente, embora a
ultrassonografia represente uma ferramenta
diagnóstica que permite a identificação precoce de
muitas alterações no trato gastrintestinal, a maioria
dos perfis ultrassonográficos carece de especificidade;
com isso, há necessidade de obtenção de amostra
para biopsia antes da formulação de diagnóstico
definitivo. A orientação ultrassonográfica é um recurso
fundamental para a aquisição de aspirados com
agulha fina e amostra de biopsia tecidual, utilizando
técnica minimamente invasiva. As vantagens desses
procedimentos estão no fato de eles não serem
altamente invasivos, permitirem a aquisição de
amostras de boa qualidade e possibilitarem o alcance
de estruturas situadas profundamente que, sob outras
condições, poderiam ser alcançadas apenas por meio
de manobras cirúrgicas mais complexas e arriscadas.
Em suma, a pesquisa ultrassonográfica é uma
ferramenta diagnóstica válida para exame do trato
gastrintestinal. Esse método, no entanto, não pode
substituir totalmente outras técnicas de diagnóstico
3/ Investigação
endoscópica
A endoscopia era considerada como um exame
complementar aos métodos diagnósticos já descritos
anteriormente; hoje em dia, entretanto, é provável que
ela seja a ferramenta mais importante disponível para
avaliação de doenças do estômago e intestino. Isso é
particularmente verdadeiro em relação aos casos
subagudos e crônicos.
Há inúmeros sinais clínicos atribuíveis às doenças do
trato gastrintestinal superior e inferior, que devem
induzir ao uso de exame endoscópico. De modo geral,
no entanto, esse exame é indicado, sobretudo, em
casos de vômito persistente (que persiste por mais de
3 a 5 dias sem melhora apesar do tratamento
empírico), vômito recorrente, diarreia crônica (que dura
2 semanas ou mais sem resposta ao tratamento
empírico ou ao manejo nutricional), perda de peso,
anorexia, tenesmo, hematêmese ou hematoquezia
(Jones, 1997).
É essencial descartar doenças metabólicas,
endócrinas e parasitárias, bem como problemas que,
embora não se originem dentro do trato
gastrintestinal, podem produzir um ou mais dos
sintomas listados anteriormente. Por essa razão, os
exames laboratoriais, a avaliação radiológica e, se
apropriada, a avaliação ultrassonográfica são
necessárias antes da endoscopia.
A endoscopia tem a vantagem indubitável de permitir
não somente o exame visual da superfície mucosa
36
Focus_SPECIAL_GI_port:01focuschatp0108FR 12/05/11 15:37 Página 37
e m
d i s t ú r b i o s
g a s t r i n t e s t i n a i s
n o
c ã o
7.
8.
9.
10.
Figura 7. Irregularidades difusas da mucosa duodenal; o diagnóstico
histológico era linfangiectasia.
Figura 8. Prolapso duodeno-gástrico em cão com vômito crônico.
Figura 9. Dilatação de estenose esofágica com balão.
Figura 10. Amplo corpo estranho esofágico (osso).
(Figura 7) mas também a amostragem direcionada,
com obtenção de inúmeras amostras para avaliação
citológica e histológica via instrumentos adequados de
biopsia. É possível examinar grande parte do sistema
gastrintestinal, excluindo o jejuno e parte do íleo.
Sempre é recomendável a obtenção de amostras,
mesmo quando não se observa nenhuma alteração na
mucosa, pois a ausência de lesões macroscópicas não
descarta a presença de modificações estruturais ou
doenças infiltrativas, que podem ser detectadas
somente via métodos histológicos.
Os exames endoscópicos possibilitam a identificação e
a localização precisas das lesões que nem sempre
podem ser identificadas por métodos radiológicos ou
ultrassonográficos, como pólipos, pequenas úlceras ou
erosões superficiais. Além disso, a endoscopia permite
a identificação de alterações ou anormalidades difíceis
de localizar com precisão por meio de outros exames
— como divertículos, hérnia de hiato, prolapso
gástrico e esofágico ou prolapso duodeno-gástrico
(Figure 8).
A endoscopia também pode ser utilizada para efetuar
muitos procedimentos terapêuticos, difíceis de serem
realizados por outros métodos. Isso pode incluir
dilatação de estenoses do esôfago dilatation of
stenoses of the oesophagus (Figura 9) ou do cólon,
remoção de corpos estranhos do estômago e esôfago
(Figura 10), posicionamento de tubos de alimentação
e retirada de lesões pedunculadas, como pólipos.
A principal desvantagem dessa técnica está na
necessidade de anestesia geral; dessa forma, a
principal contraindicação refere-se ao procedimento
anestésico em vigor. Também é preciso lembrar que os
exames endoscópicos não devem ser efetuados na
suspeita de perfuração do trato gastrintestinal.
Existem inúmeros modelos diferentes de endoscópios
37
© Davide De Lorenzi
A r m a d i l h a s
Focus_SPECIAL_GI_port:01focuschatp0108FR 12/05/11 15:37 Página 38
A r m a d i l h a s
e m
d i s t ú r b i o s
disponíveis, mas eles variam em termos de
adequabilidade para avaliação do trato digestório
superior e inferior. Esse método de diagnóstico por
imagem requer endoscópio longo e flexível, que
permite a movimentação da ponta do instrumento em
todas as direções — uma característica essencial para
explorar o trato gastrintestinal e obter amostras de
biopsia das várias áreas dos órgãos examinados. O
endoscópio também deve ter a capacidade de insuflar
ar, irrigar com água, bem como aspirar gases e
líquidos, permitindo com isso a distensão adequada
das vísceras e a limpeza da ponta do endoscópio
durante o curso do exame.
Endoscópios com diâmetro externo entre 8,0 e 9,5 mm
podem ser usados para a grande maioria dos pacientes
examinados, embora a manobra seja mais fácil com
instrumentos de diâmetro menor (7,8 mm) para cães
com menos de 5 kg. O ângulo de visão frontal não deve
ser menor que 100°, enquanto a amplitude de
movimento da ponta do endoscópio deve ser de 210°
para cima, 90° para baixo e 100° para qualquer um dos
lados. Um aspecto muito importante é o diâmetro do
canal de trabalho, pois isso dita o tamanho da pinça
que pode ser utilizada e, consequentemente, a
qualidade das amostras de biopsia que são obtidas.
Um canal de trabalho com diâmetro entre 2,0 e 2,8 mm
possibilita a obtenção de amostras de qualidade
adequada; contudo, essa dimensão do canal está
disponível apenas nos endoscópios de diâmetro maior.
Por último, o comprimento do endoscópio é um fator
limitante significativo para a exploração das partes
mais profundas do intestino. Endoscópios com menos
de 100 cm de comprimento são inadequados, pois é
g a s t r i n t e s t i n a i s
n o
c ã o
impossível examinar o duodeno de animais de médio a
grande porte.
Apesar de não ser essencial, a possibilidade de
conectar alguma câmera de vídeo à lente ocular e
visualizar as várias etapas do exame em um display
visual de alta definição deve ser considerada, pois isso
possibilita a visualização mais precisa do aspecto da
mucosa. Além disso, o operador pode trabalhar em
uma posição mais fácil e mais confortável, os
procedimentos podem ser compartilhados com
assistentes e colegas, e ainda as imagens e os
materiais não editados do vídeo podem ser coletados e
registrados.
Os pacientes submetidos à avaliação endoscópica do
trato digestório superior e inferior devem ser
adequadamente preparados (ver Quadro 2). Sem o
preparo adequado, será impossível explorar muitas
áreas em virtude da presença de alimentos, sucos
gástricos, fezes e (possivelmente) meios de contraste à
base de bário.
Como já foi enfatizada, uma das principais vantagens
do exame endoscópico está na possibilidade de
obtenção de amostras de biopsia, ao mesmo tempo
em que se visualiza diretamente o procedimento. É
essencial a realização de técnica satisfatória para
aquisição de amostras adequadas e significativas.
A pinça de biopsia utilizada deve ser a maior possível e
compatível com o diâmetro do canal de trabalho do
instrumento. Aquelas pinças de preensão tipo colher,
com colheres ovais fenestradas são as mais eficientes,
pois podem ser usadas para obter amostras em uma
profundidade maior que os instrumentos com colheres
arredondadas; além disso, o orifício evita a trituração e
Quadro 2. Indicações para o preparo de avaliação endoscópica do sistema digestório superior e inferior
Esofagogastroduodenoscopia:
• Suspender a alimentação por, no mínimo, 18 horas antes da endoscopia
• Suspender a ingestão de água por, no mínimo, 4 horas antes da endoscopia
• Não efetuar a endoscopia em menos de 24 horas após a administração de bário
• Remover a atropina e os opioides (o butorfanol, no entanto, é aceitável) do protocolo de anestesia, pois esses agentes aumentam
o tônus do esfíncter pilórico e dificultam a passagem do instrumento
• Anestesia geral, sempre com o paciente intubado e espéculo oral no lugar
Retocolonoileoscopia:
• Suspender a alimentação por, no mínimo, 36 horas (de preferência 48 horas) antes da endoscopia
• Administrar algum laxante osmótico por meio de sonda orogástrica (na dose de 25-30 mL/kg) cerca de 24 horas antes da endoscopia
• Administrar enema com água morna 24 horas e, em seguida, 3-4 horas antes da endoscopia, utilizando aproximadamente 20
mL/kg
• Como regra, repetir os enemas até que a água expelida esteja limpa e/ou clara. Pode ser administrado outro enema sob anestesia
geral, se houver necessidade.
38
Focus_SPECIAL_GI_port:01focuschatp0108FR 12/05/11 15:37 Página 39
e m
d i s t ú r b i o s
g a s t r i n t e s t i n a i s
n o
c ã o
© Davide De Lorenzi
A r m a d i l h a s
Figuras 11. A pinça de biopsia deve ser direcionada perpendicularmente à
superfície mucosa a fim de obter amostras adequadas para histopatologia.
a deformação da amostra. As pinças de biopsia com
garras serrilhadas também funcionam bem.
As biopsias devem ser obtidas toda vez que o paciente
for submetido à endoscopia, independentemente da
aparente normalidade endoscópica das áreas
examinadas. Há relatos de que a mucosa intestinal
pode aparecer normal em muitos distúrbios
inflamatórios e neoplásicos; por outro lado, tecidos
com aspecto anormal podem estar normais ao exame
histológico em até 30% dos casos.
Ao se obter as amostras de biopsia, é imprescindível
que a pinça seja direcionada perpendicularmente ao
tecido; este, por sua vez, deve ser preendido e depois
tracionado até se desprenderdetaches (Figuras 11).
Se a pinça não estiver totalmente perpendicular, ela
pode deslizar sobre o tecido e não permitir a obtenção
de amostras adequadas.
É mais fácil obter amostras de biopsia se o órgão for
desinflado ou esvaziado, pois isso reduz a tensão no
lúmen, possibilitando a preensão do tecido. Devem ser
obtidas, no mínimo, 10 amostras de cada órgão
examinado (estômago, duodeno, cólon e íleo).
4/ Tomografia
A TC é uma ferramenta muito útil para diagnosticar
doenças abdominais de todos os tipos no homem e,
embora seu uso ainda seja limitado na medicina
veterinária, não há razão pela qual essa tecnologia não
deva se tornar um exame de rotina juntamente com a
ultrassonografia e a endoscopia. A TC possibilita a
superação das limitações descritas anteriormente para
os exames radiográficos. Mais especificamente:
•A TC produz excelente resolução de baixo contraste
(Figuras 12), pelo uso de feixe altamente colimado, o
que gera corte transversal de imagem do paciente; são
utilizados detectores especiais para certificar que a
radiação necessária penetre através do corte a ser
examinado.
•Além disso, a TC permite a alteração da largura e do
comprimento da janela da imagem, fazendo com que a
escala de contraste seja modificada e adaptada às
necessidades do operador.
•As novas e rápidas técnicas de varredura, como TC
espiral, possibilitam a avaliação excepcionalmente
rápida do animal, adquirindo imagens durante o breve
período ao término da expiração. Isso significa uma
b.
Figuras 12a. Projeções sagitais
da varredura por TC
contrastada, exibindo
espessamento focal da parede
lisa do jejuno. b. Aspecto
macroscópico da lesão da
figura 12a. O diagnóstico
histológico era de um
“adenocarcinoma bemdiferenciado”.
© Davide De Lorenzi
a.
computadorizada (TC)
39
Focus_SPECIAL_GI_port:01focuschatp0108FR 12/05/11 15:37 Página 40
A r m a d i l h a s
e m
d i s t ú r b i o s
g a s t r i n t e s t i n a i s
n o
c ã o
> Perguntas a Todd Tams
Conseguimos diagnosticar os distúrbios gastrintestinais (GI) com precisão sem endoscópio?
Todd Tams
Muitos distúrbios GI podem ser diagnosticados sem o auxíleo de endoscópio; contudo, alguns distúrbios necessitam de
visualização direta ou análise histológica da mucosa para estabelecimento de diagnóstico preciso. Na maioria desses
casos, a endoscopia representa um método minimamente invasivo para a formulação do diagnóstico correto. A maior
parte dos proprietários de pequenos animais prefere que seu pet seja submetido a procedimento relativamente
descomplicado e bastante seguro (ou seja, endoscopia GI flexível) a ter de optar pela via cirúrgica muito mais invasiva
na obtenção de amostras teciduais para biopsia.
Os distúrbios que necessitam de biopsia para o diagnóstico definitivo incluem distúrbios inflamatórios, neoplásicos e,
em alguns casos, fúngicos. A endoscopia não só fornece informações valiosas na suspeita de esofagite, lesão de
mucosa por medicamentos (como agentes anti-inflamatórios não esteroidais) ou anormalidades anatômicas nas áreas
do antro gástrico e pilórico, mas também ajuda a descartar distúrbios infiltrativos quando as biopsias são consideradas
normais ou apenas levemente anormais. Seguramente, os achados negativos de biopsia são boas notícias e ajudam a
redirecionar os esforços terapêuticos do clínico, por exemplo, para distúrbios de motilidade ou reações adversas a
alimentos.
Apesar das tremendas vantagens diagnósticas oferecidas pela endoscopia, essa técnica ainda é melhor utilizada pelo
clínico como procedimento adjuvante na avaliação de doença GI. Em grande parte dos casos, a endoscopia é realizada
assim que a avaliação do histórico, o exame físico completo, os exames diagnósticos basais e os ensaios alimentares
apropriados, se indicados, forem concluídos. Em casos brandos, também é totalmente apropriado tentar a terapia
empírica nos estágios iniciais da doença antes de embarcar em procedimentos diagnósticos mais caros. Os distúrbios
prontamente diagnosticados por exames laboratoriais simples incluem parasitas GI, enterotoxicose por Clostridium
perfringens, insuficiência pancreática exócrina, distúrbios metabólicos, hipertireoidismo, etc. A radiografia e a
ultrassonografia são exames úteis para avaliação de uma série de distúrbios do trato alimentar. Notar que alguns
exames, particularmente a análise das fezes, precisam ser repetidos uma ou mais vezes antes que o diagnóstico possa
ser feito.
A endoscopia deve ser considerada em qualquer cão ou gato com vômito ou diarreia intermitente, cujos exames basais
falharam em fornecer o diagnóstico e cuja terapia se mostrou ineficaz. O ponto-chave recomendado por mim é que o
clínico não deve esperar muito tempo para recomendar a endoscopia. Muitas vezes, espera-se que o paciente tenha
sinais cronicamente persistentes por vários meses antes que se recomende a realização de endoscopia ou laparotomia
exploratória. Com a pronta disponibilidade dessa versátil ferramenta, deve haver pouca hesitação para obter o exame
visual do esôfago, estômago, intestino delgado superior e, se necessário, de todo o cólon e, com certeza, a endoscopia
frequentemente leva à determinação mais precoce do melhor curso terapêutico geral.
?
40
Focus_SPECIAL_GI_port:01focuschatp0108FR 12/05/11 15:37 Página 41
A r m a d i l h a s
e m
d i s t ú r b i o s
a.
g a s t r i n t e s t i n a i s
n o
c ã o
c.
© Davide De Lorenzi
b.
Figuras 13. Varredura por TC contrastada em cão, revelando espessamento
circunferencial da parede gástrica (adenocarcinoma gástrico).
a. Projeção dorsal
b. Projeção sagital
c. Reconstrução tridimensional (Renderização de Volume)
melhoria na visualização tridimensional, permitindo a
reformatação da imagem multiplanar e o uso de novas
aplicações, como visualização contínua, angiografia
por TC, imagem virtual da realidade e endoscopia por
TC.
•As imagens digitais adquiridas dessa forma podem
ser processadas a fim de aumentar a quantidade de
informações contidas dentro delas para uma
compreensão perfeita do formato e da estrutura das
lesões (Figures 13).
A administração de meio de contraste diluído (p. ex.,
diatrizoato de meglumina diluído em água na
proporção de 1:25) por via oral confere a opacificação
de todo o trato gastrintestinal, permitindo a
diferenciação segura entre as alças intestinais
homogêneas preenchidas de líquido e os abscessos ou
massas. A dilatação simultânea do estômago e
intestino com ar aumentará ainda mais a qualidade
das imagens obtidas.
a.
c.
d.
© Davide De Lorenzi
b.
Figuras 14. O mesmo problema (adenocarcinoma gástrico) observado por diferentes técnicas diagnósticas:
a. Radiografia abdominal; b. Ultrassonografia; c. Endoscopia; d. Tomografia computadorizada.
41
Focus_SPECIAL_GI_port:01focuschatp0108FR 12/05/11 15:37 Página 42
A r m a d i l h a s
e m
d i s t ú r b i o s
g a s t r i n t e s t i n a i s
n o
c ã o
Tabela 1. Tabela sinóptica das técnicas diagnósticas descritas no capítulo 4.
Anestesia
Detecção de corpo
estranho
Avaliação da motilidade
Espessura do intestino
Coleta de amostra
Riscos para seres humanos
Detecção de problemas
extraintestinais
Aspectos da mucosa
Remoção de corpo
estranho
Radiografia
Não
Sim, apenas se radiopaco
Ultrassonografia
Não
Sim, mas depende da localização
Endoscopia
Sim
Sim, mas depende da
localização
Ocasionalmente útil
Não
Não
Sim
Sim, dependendo da natureza
da lesão
Não
Não
Sim
Sim
Sim, se houver a presença de massa
Não
Sim
Sim, mas imprecisa
Não
Sim, excelente
Não
Não
Não
Não
Sim
Sim
> Conclusão
>
Conforme foi descrito anteriormente, existem inúmeras técnicas de diagnóstico por imagem para avaliação e diagnóstico
aprofundados de doenças do trato gastrintestinal. Cada uma delas possui indicações e desvantagens específicas, dependendo do
tipo de tecnologia utilizada (Tabela 1).
O conhecimento exato do que pode ou não ser obtido com as diferentes modalidades de equipamentos de imagem possibilita o
emprego bem-fundamentado e excelente dos recursos diagnósticos que se encontram à disposição da medicina veterinária
atualmente (Figuras 14). Por último, a inclusão dessas tecnologias em um procedimento diagnóstico rigoroso e padronizado
permitirá a obtenção da maior quantidade possível de informações, sem recorrer a elas quando seu uso não for realmente indicado.
42
Focus_SPECIAL_GI_port:01focuschatp0108FR 12/05/11 15:37 Página 43
A r m a d i l h a s
e m
d i s t ú r b i o s
g a s t r i n t e s t i n a i s
n o
c ã o
5. Os principais erros
cometidos no tratamento de
distúrbios gastrintestinais
caninos
*
> Resumo
Um clínico criterioso deve abordar todo caso com objetivos claros para garantir o melhor tratamento
do problema clínico.
• Iniciar os exames complementares imediatamente em casos apropriados.
• Evitar o uso de vários tratamentos sintomáticos sucessivos; se os sinais clínicos persistirem, é
desejável obter o diagnóstico etiológico o mais rápido possível.
• Se o tratamento prolongado for prescrito, abordar claramente com os proprietários o motivo do
tratamento e suas limitações terapêuticas. Informá-los sobre qualquer efeito colateral possível e o
prognóstico do distúrbio de seu pet.
• Se o ajuste da dieta for essencial, lembrar que a comida não é medicamento e, portanto, não
substitui o tratamento médico, nos casos em que houver indicação.
Introdução
O tratamento médico de distúrbios digestivos em
cães pode ser complexo. O proprietário de animal
doente espera uma melhora clínica rápida e
definitiva ao término do tratamento, particularmente
se os sintomas estiverem evoluindo ao longo do
tempo e/ou se o estado geral do animal estiver
piorando.
2. esquecer de realizar algum exame complementar
ou cometer um erro na interpretação dos resultados
do exame
3. não respeitar certas regras ao colocar o
tratamento em prática
4. não se comunicar com o proprietário em casos
que necessitam de esquemas terapêuticos
complexos ou prolongados.
1/ Sinais e lembretes
importantes
Pontos-chave
Os principais erros possivelmente cometidos
durante o tratamento de distúrbios gastrintestinais
podem resultar de atitudes, como:
1. não identificar certos sinais significativos
A presença de certos sinais clínicos deve
imediatamente induzir o clínico a realizar exames
complementares apropriados em ordem de
prioridade para possibilitar a prescrição de
43
Focus_SPECIAL_GI_port:01focuschatp0108FR 12/05/11 15:37 Página 44
A r m a d i l h a s
e m
d i s t ú r b i o s
tratamento para a causa da doença e não
simplesmente para os sintomas; isso sempre deve
ser o principal objetivo de um clínico competente.
g a s t r i n t e s t i n a i s
n o
c ã o
exemplo, diferentes estudos epidemiológicos
demonstraram a predisposição racial a carcinoma
gástrico (Estrada, 1997): este é particularmente o
caso de Pastor Belga, Collie e Chow Chow. Nessas
raças, o exame em busca dessas lesões via
gastroscopia deve ser sugerido em um estágio
precoce se o vômito não parar dentro de alguns dias
do início do tratamento antiemético.
A) Em caso de vômito ou diarreia
Dois quadros são possíveis:
Se o vômito ou a diarreia forem “agudos”, a
presença de um ou mais dos fatores expostos a
seguir pode indicar a necessidade de hospitalização
do animal ou a instituição imediata de avaliação
para determinar a causa do problema.
• Se o paciente for muito pequeno.
• Se o estado geral do animal se deteriorar.
• Se houver desidratação, febre ou hipotermia,
hematêmese ou melena.
• Se houver dor abdominal, vômito ou diarreia
incontrolável.
• Se houver risco de possível agravamento do
distúrbio ou caso pareça possível a presença de
outros distúrbios.
B) Em caso de constipação ou
hematoquezia
A constipação é bem menos comum em cães do que
em gatos. Por definição, esse sintoma consiste em
declínio do ato de defecar com a evacuação de fezes
secas ou muito pequenas. A causa de constipação
deve ser rapidamente identificada se o paciente
apresentado tiver dificuldade de eliminação das
fezes sem qualquer antecedente específico e se
estiver recebendo alguma dieta balanceada (e se,
durante a consulta, o exame retal possibilitar a
exclusão de hiperplasia prostática no cão macho). A
passagem das fezes pode ser dolorosa (disquezia) e
isto é um sinal clínico de “alerta” para o veterinário.
Hematoquezia é a presença de sangue em fezes
formadas. As fezes podem ou não ter diâmetro
reduzido. Esses traços de sangue costumam ser
observados como vestígios/rastros lineares ou
manchas vermelhas, mas sua aparência sempre
deve induzir à avaliação do veterinário, antes de
qualquer tratamento, na busca de alguma lesão
endoluminal responsável pelo sangramento. Após
preparo colônico adequado, pode-se efetuar a
colonoscopia, inspecionando todo o cólon, inclusive,
se possível, a válvula ileocecocólica (Tams, 1996).
As biopsias realizadas em intervalos durante o curso
do exame devem permitir a identificação histológica
de quaisquer lesões visualizadas: notar que, em
alguns casos, não é tarefa fácil a diferenciação
macroscópica entre lesão benigna e tumor
canceroso (De Novo, 2000).
Como os tumores colorretais assumem uma
variedade de aspectos macroscópicos, vale à pena
destacar os seguintes pontos:
• Pólipos pedunculados podem ser encontrados no
cólon distal, especificamente dentro dos 10
centímetros do ânus e nas pregas da margem anal
(Figura 1). Algumas vezes, talvez sejam
Se o vômito ou a diarreia forem “crônicos” (ou seja,
presença dos sinais clínicos por mais de 1 semana
ou ocorrência de forma intermitente ou recorrente),
é imperativa a realização de exames
complementares antes de iniciar qualquer
tratamento.
O exame clínico é parte essencial do processo de
obtenção do diagnóstico. É necessário que esse
exame seja preciso e meticuloso, pois
frequentemente existem inúmeros e diversos
diagnósticos diferenciais possíveis. O registro do
histórico do caso deve ser meticuloso também, mas
os quadros de perda de peso progressiva, mudanças
de comportamento e/ou alteração do apetite são,
sem exceção, sinais clínicos que devem ser levados
em conta.
Após descartar causas metabólicas e distúrbios
digestivos, é recomendável a execução de uma
sequência de exames complementares: análises
hematobioquímicas,
exames
radiográficos,
ultrassonografia abdominal, endoscopia, etc. A
sequência dos exames depende muito do histórico
do caso e dos fatos relatados pelo proprietário, bem
como da idade, raça e estilo de vida do animal. Os
exames clínicos e complementares devem ser
especificamente adaptados a cada caso. Por
44
Focus_SPECIAL_GI_port:01focuschatp0108FR 12/05/11 15:37 Página 45
© Valérie Freiche
A r m a d i l h a s
e m
d i s t ú r b i o s
g a s t r i n t e s t i n a i s
n o
c ã o
ensaios seriados de amilase e lipase diariamente
por vários dias. As amostras devem ser enviadas,
sempre que possível, a um laboratório veterinário de
confiança.
Figura 1. A inversão
da ponta do
endoscópio durante
a colonoscopia
demonstrou a
presença de lesões
na face interna da
margem anal.
Ao realizar uma avaliação hematobioquímica em cão
com vômito, não omitir a avaliação eletrolítica:
esses
resultados
ajudam
a
excluir
hipoadrenocorticismo (o teste de estimulação com
ACTH também pode ser feito, se requerido) e são
necessários para garantir a reidratação adequada do
animal durante o período de hospitalização.
Se houver necessidade de coproscopia ou
coprocultura parasitária, esses exames deverão ser
idealmente realizados por laboratório veterinário
confiável; algumas vezes, há necessidade de várias
amostras para fazer o diagnóstico (p. ex., na
pesquisa de protozoários tipo Giardia). Para mais
informações, recorra à página 27.
A interpretação dos resultados laboratoriais não
deve suscitar tratamentos em excesso; infelizmente,
isso acontece com muita frequência na clínica geral.
Por exemplo:
1. Diarreia fúngica é rara; se detectada nas fezes, a
Candida geralmente se deve à colonização
secundária oportunista após alteração na microbiota
bacteriana digestiva (p. ex., pós-antibioticoterapia).
Notar que o tratamento antifúngico não deve ser
administrado sem uma boa razão, particularmente
se a causa da diarreia não foi determinada.
2. Diarreia bacteriana, no sentido mais estrito,
também é muito rara na clínica. Muitas bactérias
potencialmente enteropatogênicas pertencem à
microbiota intestinal endógena. A administração de
antibióticos pode ser nociva, ou até mesmo
contraindicada, se esses agentes terapêuticos
forem administrados de forma empírica durante
algum episódio de diarreia no cão. O uso de
antibióticos deve seguir regras sistemáticas e muito
rigorosas (Marks, 2000). As bactérias mais
frequentemente envolvidas são: Clostridium
perfringens ou difficile, Escherichia coli,
Campylobacter spp., Salmonella, e Yersinia
enterolitica. Portanto, a identificação de colônias de
E. coli após análise bacteriológica das fezes pode
ser “normal” e não necessitar de antibioticoterapia
oral se a colônia não apresentar propriedades
identificados múltiplos
pólipos. Esses pólipos são frequentemente
benignos, mas a análise histológica pode revelar o
aspecto carcinomatoso do ápice da lesão, o que
justificaria a radioterapia complementar.
• Massas endoluminais diferem bastante em
relação ao aspecto; as lesões podem variar em
termos de grau de circunscrição e quantidade
projetada no lúmen, mas talvez sejam de natureza
friável, hemorrágica ou ulcerada. O prognóstico das
massas endoluminais é pior que o dos pólipos, pois
essas massas são anulares, não proliferativas e
levam à rigidez parietal significativa.
2/ Escolha cuidadosa dos
exames complementares e
interpretação precisa dos
resultados
Se não houver resposta terapêutica, o diagnóstico
sempre deve ser questionado: uma possível causa
de ausência de resposta ao tratamento pode ser a
negligência de algum exame complementar ou o
erro de interpretação de alguns resultados. Aqui,
estão alguns exemplos:
Não descartar causas metabólicas de distúrbios
digestivos. Em animais jovens, algumas anomalias
congênitas talvez causem sinais clínicos, que podem
ser erroneamente atribuídos a “gastrenterite”:
•
Displasia renal em Pastor Alemão pode
levar à insuficiência renal precoce, mas os sintomas
disso nem sempre são específicos.
•
Desvios portossistêmicos podem causar
sinais gastrintestinais em filhotes caninos;
considerar os testes de ácidos biliares pré e pósprandiais se houver suspeita.
•
Na suspeita de pancreatite, realizar
45
Focus_SPECIAL_GI_port:01focuschatp0108FR 12/05/11 15:37 Página 46
A r m a d i l h a s
e m
d i s t ú r b i o s
enteropatogênicas conhecidas; a tipagem da E. coli
nem sempre é fácil. No mesmo sentido, Cl.
perfringens pode ser identificado em mais de 80%
das análises fecais tanto em cães doentes como nos
assintomáticos.
3. Salmonelose clínica é muito rara em carnívoros
domésticos e parece ser a causa de menos de 2%
das síndromes diarreicas (Marks, 2000).
g a s t r i n t e s t i n a i s
n o
c ã o
exames radiográficos sejam adaptados e a posição
do animal esteja correta. Sempre é desejável obter
todas as radiografias em duas projeções: lateral e
ventrodorsal. Os estudos com contraste são
solicitados em um número cada vez mais limitado de
casos; isso se deve principalmente ao
desenvolvimento de técnicas mais eficientes de
ultrassonografia e endoscopia. Se o estudo com
bário for sugerido, no entanto, deve-se realizá-lo,
utilizando uma abordagem lógica e sistemática sob
condições ideais: isso envolve a obtenção de
radiografias nas projeções apropriadas em
intervalos regulares, a visualização de filmes
radiográficos sucessivos lado a lado para
comparação (especialmente se houver qualquer
dúvida a respeito da imagem), etc.
Uma ultrassonografia digestiva conduzida sob
condições técnicas inadequadas nem sempre
fornece os dados que seriam esperados ao utilizar
essa ferramenta diagnóstica. O treinamento da
equipe e o uso de equipamentos eficazes são
essenciais para a realização de exames de
qualidade. Contudo, as indicações e limitações
desse exame complementar — que mais
recentemente se tornou essencial para os
gastrenterologistas — devem ser conhecidas. Se
possível, o animal deve ficar em jejum 24 horas
antes do exame. Se planejada, a endoscopia deve
ocorrer após a ultrassonografia, de modo que
qualquer ar insuflado durante o curso da avaliação
não interfira na propagação do ultrassom. Algumas
partes do trato intestinal necessitam de operador
experiente para garantir a boa visualização,
especialmente a região pancreática do duodeno e
(por conta da presença de gás) o cólon maior.
Caso se realizem biopsias guiadas por ultrassom,
deve-se ter em mente que os resultados obtidos
podem não ser representativos da lesão, pois nem
sempre é fácil mirar a lesão com precisão durante a
ultrassonografia.
Durante a endoscopia, conforme previamente
observado, é preciso ter cuidado para não anunciar o
diagnóstico definitivo antes dos resultados da
histopatologia, mesmo se, em certos casos, o
provável problema parecer óbvio. O aspecto
macroscópico de algumas lesões pode ser
enganoso, pois uma reação inflamatória grave pode
ter o mesmo aspecto que uma neoplasia. Do mesmo
A formulação de diagnóstico consistente para
diarreia de origem bacteriana, portanto, apoia-se
nos dados clínicos e epidemiológicos, nas análises
bacteriológicas e, até mesmo, nos testes de reação
em cadeia da polimerase (PCR) em casos duvidosos.
A decisão em administrar tratamento antibacteriano
em problemas gastrintestinais pode ser limitada às
principais indicações a seguir:
• Isolamento de colônia bacteriana enteropatogênica em um contexto epidemiológico específico
(pode ser interessante saber se outros animais que
convivem com o cão ou se membros da família
apresentam distúrbios digestivos simultaneamente).
• Diagnóstico da síndrome de supercrescimento
bacteriano do intestino delgado. Ainda não se sabe
se essa síndrome é de natureza primária ou
secundária. De fato, a colonização bacteriana pode
suceder inúmeros distúrbios gastrintestinais
crônicos, mas pode recorrer repetidas vezes se a
causa primária não for identificada nem tratada; isso
ocorre particularmente em casos de insuficiência
pancreática exócrina e também com doença
inflamatória intestinal crônica.
• Gastrite indutora de ulceração da parede
intestinal com agravamento do estado geral do
animal (p. ex., caso se desenvolva febre ou diarreia
hemorrágica).
• Diagnóstico de gastrite por Helicobacter, em que
a terapia tripla composta por dois antibióticos e um
antiácido pode ser apropriada se o estado clínico do
animal justificá-la. Notar, entretanto, que um
número muito grande de cães saudáveis são
portadores desse microrganismo sem exibir
quaisquer sintomas digestivos; além disso, entendese atualmente que o Helicobacter não tem o mesmo
potencial patogênico em cães como em seres
humanos.
Ao se efetuar as radiografias, é necessário que os
46
Focus_SPECIAL_GI_port:01focuschatp0108FR 12/05/11 15:37 Página 47
e m
d i s t ú r b i o s
modo, a presença de infiltração neoplásica difusa,
como linfoma, pode passar despercebida. Além
disso, há limitações topográficas relacionadas ao
porte do animal e à extensão do trato digestório, o
que pode limitar a eficácia do exame endoscópico.
As biopsias podem ser inadvertidamente obtidas
fora do local da lesão ou talvez não sejam
representativas. Três exemplos merecem ser
apontados aqui:
• No caso de úlcera gástrica, a possibilidade de
tumor subjacente deve ser considerada (úlceras
gástricas isoladas são muito raras em carnívoros
domésticos). As biopsias podem simplesmente
revelar material inflamatório e necrótico sem
demonstrar quaisquer células malignas. Essa
possibilidade justifica a obtenção de um número
maior de biopsias endoscópicas se houver suspeita
de carcinoma gástrico. Em sua forma ulcerada e
infiltrativa, observa-se espessamento da camada
parietal principal. Talvez haja necessidade de dez
biopsias para obter células cancerosas em apenas
uma ou duas das amostras (Figura 2).
• Em caso de linfoma gastrintestinal, notar que
lesões inflamatórias, difusas, relacionadas podem
coexistir com a neoplasia em muitos casos.
• No caso de neoplasia colorretal, podem ocorrer
lesões malignas esporadicamente ao longo da
extensão do intestino e, por essa razão, essas lesões
talvez não sejam identificadas apesar das inúmeras
biopsias endoscópicas. Em pacientes que se
apresentam com desconforto funcional e dor
abdominal, esses fatos justificarão algumas vezes a
avaliação ecográfica extensa e a intervenção
cirúrgica imediata se os proprietários estiverem
dispostos. É recomendável a análise cuidadosa do
tecido biopsiado e de suas margens; além disso, a
g a s t r i n t e s t i n a i s
n o
c ã o
terapia adjuvante (quimio e/ou radioterapia) deve
ser providenciada, se necessária.
Por todas essas razões, é desejável que o clínico
seja capaz de “ler nas entrelinhas” e não se
acomodar apenas com a conclusão dos resultados
histológicos. Além disso, se depois de alguns dias, o
animal não exibir melhora clínica como os exames
laboratoriais sugeririam, o veterinário não deve
hesitar em questionar o diagnóstico.
3/ Princípios básicos em
relação ao início do
tratamento médico e à
implantação de novas
medidas dietéticas
A administração de tratamento sintomático ou
etiológico deve obedecer a certos critérios,
conforme exposto a seguir:
Em caso de tratamentos sucessivos sem êxito, é
aconselhável verificar a dosagem dos diferentes
medicamentos utilizados; isso é particularmente
verdadeiro em situações de encaminhamento em
que outros tratamentos foram instituídos por
veterinários anteriores. Também é preciso resistir à
tentação de aplicar automaticamente uma dosagem
humana em cães; é necessário que alguns
medicamentos sejam prescritos em uma dose mais
baixa, e outros, em uma dose mais alta.
É imperativo que vários tratamentos sintomáticos
sucessivos contendo ingredientes ativos com
mecanismo de ação similar não sejam
administrados concomitantemente (Figura 3). Por
exemplo, se um antiemético, como a
Figura 3. Evitar o uso de diversos
medicamentos simultaneamente por causa das
possíveis interações e pelo custo como fator
limitante de obediência ao tratamento.
47
© Valérie Freiche
A r m a d i l h a s
Figura 2. Úlcera
gástrica não
neoplásica
isolada.
Focus_SPECIAL_GI_port:01focuschatp0108FR 12/05/11 15:37 Página 48
A r m a d i l h a s
e m
d i s t ú r b i o s
Figura 4. É
muito importante
conversar
com o
proprietário
ao explicar
o tratamento.
metoclopramida, foi experimentado e o estado do
animal não melhorou nem piorou, é improvável que
um segundo antiemético com o mesmo mecanismo
de ação seja bem-sucedido. Além disso, também é
indesejável, nesse tipo de situação, prescrever um
antiemético de nova geração com mecanismo de
ação distinto (por exemplo, algum antagonista
serotoninérgico, como a ondansetrona), pois é
improvável que ele seja mais bem-sucedido que o
medicamento original: o diagnóstico definitivo é
essencial nesse estágio.
É preciso ter em mente que certo número de
medicamentos possui efeitos colaterais bemconhecidos, que devem ser comentados com o
proprietário ao serem prescritos: por exemplo, a
metoclopramida tem efeitos extrapiramidais, que
podem dar origem a sintomas comportamentais
indesejáveis, como agitação ou agressão. De
tempos em tempos, também há intolerâncias
idiossincráticas, que se originam em alguns animais
e devem ser consideradas.
Não prolongar a administração de qualquer
medicamento administrado por alguns dias sem
nenhum efeito: por exemplo, se um cão diarreico for
submetido à loperamida por 3 ou 4 dias, mas a
diarreia ainda persistir, é inútil prescrever esse
tratamento em um esquema prolongado ou esperar
por seus benefícios a longo prazo. Quando um
medicamento for selecionado a partir da
farmacopeia humana (alguns medicamentos
essenciais não possuem equivalentes na medicina
veterinária), é desejável lembrar o proprietário que
os efeitos colaterais listados na bula do remédio
foram estabelecidos com base nos estudos em seres
humanos, não em cães; por exemplo: a
salazosulfapiridina, indicada em casos de colite,
g a s t r i n t e s t i n a i s
n o
c ã o
lista uma série de efeitos colaterais, que não são
provavelmente observados em cães.
Tendo iniciado o tratamento, o clínico deve estar
ciente do lapso de tempo esperado antes que uma
melhora possa ser razoavelmente esperada ou em
que momento outras avaliações são indicadas se os
sinais clínicos persistirem.
A escolha da dieta pode exercer um impacto
tremendo sobre os distúrbios gastrintestinais:
durante a consulta, é aconselhável verificar que tipo
de dieta é fornecido ao animal e checar todos os
possíveis complementos e quantidades — muitas
vezes, isso revelará que a dieta declarada é
simplesmente uma “base” a qual se adicionam
vários ingredientes. É desejável explicar ao
proprietário o que se espera de qualquer alimento
novo proposto como complemento à terapia médica.
É uma boa ideia fixar um prazo razoável de espera
por algum resultado, geralmente alguns dias e não
algumas semanas (o contrário ao que se pratica para
dietas de eliminação dermatológicas, por exemplo).
Um proprietário nunca deve ser incentivado a
acreditar que os métodos relacionados à dieta
sempre podem ser um substituto total para o
tratamento médico.
4/ Conversar com o
proprietário ao implantar
tratamento de médio ou
longo prazo
A) Qualidade do
acompanhamento clínico e
terapêutico
Na presença de doença crônica e/ou incapacitante
que requer tratamento médico por várias semanas
ou meses (p. ex., tumor gástrico ou distúrbio
inflamatório crônico grave), os proprietários podem
ter altas expectativas e/ou exigências; é prudente
ter, no local, sistemas que permitem fácil acesso
ao clínico, fazendo com que os proprietários
recebam atenção e orientação, sempre que houver
necessidade. Talvez haja necessidade, sem
exceção, de acompanhamento dos casos por
48
Focus_SPECIAL_GI_port:01focuschatp0108FR 12/05/11 15:37 Página 49
A r m a d i l h a s
e m
d i s t ú r b i o s
g a s t r i n t e s t i n a i s
telefone (Figuras 4 & 5), check-ups clínicos,
análises laboratoriais ou exames complementares
repetidos. O termo “veterinário da família” adquire
seu pleno significado por essa diligência.
n o
c ã o
Figura 5. Manter
contato periódico
com o
proprietário do
cão que esteja
sofrendo de
doença
crônica e
recebendo
tratamento
vitalício.
B) Controle de doenças
crônicas: a importância da
obediência ao tratamento
Continua sendo fundamentalmente importante
prescrever um tratamento que possa ser
obedecido: o proprietário deve ser capaz de lidar
com o nível de contenções e tratamentos impostos
em um esquema diário o máximo de tempo
possível, conforme a necessidade. Para fazer isso,
é preciso considerar um grande número de
parâmetros, mas especialmente o estilo de vida e o
temperamento do animal, bem como a motivação e
a capacidade de adaptação do proprietário. A
obediência do proprietário também pode ser um
problema a curto prazo durante o preparo do cão
para a realização de exames complementares
(como a suspensão de alimento para colonoscopia,
a adesão à dieta líquida para ultrassonografia ou
exames de sangue, etc.).
Do ponto de vista farmacológico, nem sempre é
fácil controlar os efeitos de um número
excessivamente grande de medicamentos
oferecidos de forma simultânea: sempre que
possível, é preferível usar não mais que 4 ou 5
medicamentos ao mesmo tempo.
Quando a quimioterapia for requisitada, o
proprietário deve estar plenamente ciente do
provável raio de alcance do tratamento. É de suma
importância levar o provável “tempo de sobrevida”
do animal em consideração e descrever claramente
as restrições terapêuticas e quaisquer efeitos
colaterais possíveis dos medicamentos a serem
administrados.
Certo número de doenças digestivas crônicas
necessita da administração prolongada de
corticosteroides ou imunomoduladores. Por outro
lado, o uso desses agentes terapêuticos deve ser
racionalizado e restringido, mas é recomendável
que a necessidade de recorrer a eles seja abordada
com o proprietário, de modo que a obediência ao
tratamento se torne possível.
O mesmo é verdadeiro ao prescrever tratamento
médico vitalício em caso de insuficiência
pancreática exócrina diagnosticada em cão. Como
a cura permanente é impossível, a resposta
terapêutica pode ser instável, com surtos repetidos
de diarreia secundária ao supercrescimento
bacteriano do intestino delgado. O custo do
tratamento a longo prazo, particularmente em cães
de grande porte, pode induzir o proprietário a optar
pela eutanásia.
Conclusão
O diagnóstico precoce de qualquer paciente está
intimamente ligado à abordagem metódica e
clínica meticulosa, bem como à escolha e
sequência cuidadosas de testes diagnósticos. A
interpretação correta dos resultados desses
exames governará a seleção de tratamento médico
adequado. O tratamento deve ser feito com o
consentimento dos proprietários (ou seja, com a
assinatura do termo de consentimento informado),
notificando-os da duração, do custo e dos possíveis
efeitos colaterais dos agentes terapêuticos
escolhidos. As qualidades e habilidades pessoais
do clínico são particularmente cruciais ao lidar com
distúrbios digestivos crônicos, extremamente
comuns em carnívoros domésticos.
49
Focus_SPECIAL_GI_port:01focuschatp0108FR 12/05/11 15:37 Página 50
A r m a d i l h a s
e m
d i s t ú r b i o s
g a s t r i n t e s t i n a i s
n o
c ã o
> Perguntas a Todd Tams
Quando o tratamento empírico é aceitável e quando ele não é aceitável?
Todd Tams
Os distúrbios gastrintestinais são uma das razões mais comuns que levam os cães aos veterinários em busca de cuidados, mas o
tratamento sintomático é comumente prescrito. Embora, em muitos casos, a terapia empírica possa ser apropriada, deve-se enfatizar
que as melhores decisões terapêuticas são tomadas com base nos princípios básicos da medicina; ou seja, sempre há necessidade de
obter histórico preciso e realizar exame físico completo. Como especialista em gastrenterologia, tenho visto muitos casos ao longo dos
anos em que expressivos erros foram cometidos no tratamento dos casos como resultado de revisão inadequada do histórico do
paciente e dos sinais clínicos mais significativos, bem como em consequência da seleção de medicamentos inapropriados para o
distúrbio real do paciente. Além disso, a falha na execução de um conjunto adequado de exames básicos, que poderia ajudar no
estabelecimento do diagnóstico, pode retardar a instituição do melhor curso terapêutico para o paciente.
São exemplos de possíveis erros: tratar um paciente para vômito quando, na verdade, ele está regurgitando por algum distúrbio
esofágico, como megaesôfago; utilizar agentes anticolinérgicos para diarreia do intestino grosso quando a terapia anti-inflamatória para
colite aguda seria mais apropriada; não examinar o animal adequadamente em busca de Giardia e outros parasitas intestinais; e tentar
vários cursos terapêuticos empíricos em cão acometido por enteropatia com perda proteica enquanto se falha na triagem conveniente
com hemograma completo e perfil bioquímico — um paciente com pan-hipoproteinemia sofre de enteropatia moderada a grave e, por
essa razão, devem ser tomadas medidas para estabelecer o diagnóstico definitivo o mais cedo possível.
Em geral, a terapia empírica é apropriada em animais levemente afetados com normorexia (manutenção de apetite satisfatório),
exibição de sinais clínicos apenas a curto prazo e sem perda de peso. Os exemplos de esquemas empíricos razoáveis incluem
metronidazol e/ou sulfassalazina para colite aguda; suspensão de alimento por 12-24 horas e prescrição de bloqueador dos receptores
histaminérgicos H2 em animais com vômito agudo; ensaio nutricional com dieta constituída por nova fonte proteica durante 2-3
semanas se a sensibilidade alimentar for considerada uma possibilidade; suplementação de fibras alimentares para diarreia do intestino
grosso ou constipação; administração de agente anti-helmíntico de amplo espectro na suspeita de parasitas intestinais, ainda que os
exames fecais de triagem sejam negativos; e curso de antibioticoterapia de 2 semanas para possível diarreia responsiva a antibióticos
em cão.
O uso contínuo de terapia empírica não é aceitável em pacientes com inapetência (falta de apetite), letargia ou perda de peso; ou
naqueles que ainda se encontram aparentemente em bom estado geral, mas apresentam vômito ou diarreia contínuo. Esses animais
merecem o benefício proporcionado pelos testes diagnósticos, que ajudarão a determinar o que está errado e, como veterinários,
precisamos dar o melhor de si para defender nossos pacientes, convencendo nossos clientes de que a coisa certa a ser feita é tomar
medidas apropriadas para formular o diagnóstico correto. Isso também significa que, em alguns casos, haverá necessidade de
encaminhamento para algum especialista por parte dos clínicos gerais, para acelerar o diagnóstico pelo uso de ferramentas
diagnósticas especiais.
?
?
> Perguntas a Denise Elliott
Qual o interesse envolvido no fornecimento de alto teor de gordura no controle dietético de distúrbios GI?
A gordura é o mais altamente digerível de todos os nutrientes, com valores de digestibilidade superiores a 90%. Dietas ricas em gordura
são calóricas (ou seja, densas energeticamente), reduzindo o volume de alimento consumido em cada refeição. Além disso, a gordura
retardará o esvaziamento gástrico e prolongará a digestão. Cada uma dessas características da gordura pode ser benéfica para algumas
formas de enteropatias. De fato, também foi demonstrado que a gordura seja bem tolerada por cães com uma variedade de distúrbios
gastrintestinais (doença inflamatória intestinal, gastrite aguda ou crônica) e exerça efeito benéfico sobre o apetite, o ganho de peso e a
Denise Elliott resolução dos sinais clínicos de vômito e diarreia.
E quanto ao baixo nível de gordura no controle dietético de distúrbios GI?
Dos nutrientes que fornecem de energia, a gordura é o nutriente de digestão mais difícil, pois requer a interação entre intestino, fígado
e pâncreas. A deficiência de enzimas pancreáticas não só prejudica a digestão, mas também resulta em má-absorção de gordura,
proteína e carboidrato. Contudo, a digestão de gordura é mais gravemente comprometida desde que as lipases estejam ausentes do
conjunto normal de enzimas da borda intestinal em escova. As bactérias presentes no trato intestinal são capazes de metabolizar a
gordura não digerida em hidroxi-ácidos graxos, o que leva à diarreia secretória no intestino grosso. As bactérias também promovem
desconjugação dos ácidos biliares, prejudicando ainda mais os processos de digestão e absorção de gordura. Por esse motivo, a
restrição de gordura é benéfica para distúrbios em que esse nutriente pode se tornar disponível ao metabolismo microbiano, por
exemplo, na síndrome de má-absorção ou supercrescimento bacteriano do intestino delgado.
Ao contrário dos aminoácidos e monossacarídeos, absorvidos diretamente pela corrente sanguínea, a gordura é levada dos enterócitos
para os vasos quilíferos e transportada para a circulação sistêmica via vasos linfáticos mesentéricos e pelo ducto torácico. O quadro de
linfangiectasia prejudicará o transporte de gordura. Portanto, a restrição de gordura da dieta é claramente indicada para o tratamento de
linfangiectasia.
50
Focus_SPECIAL_GI_port:01focuschatp0108FR 12/05/11 15:37 Página 51
A r m a d i l h a s
e m
d i s t ú r b i o s
g a s t r i n t e s t i n a i s
n o
c ã o
6. Perguntas frequentes e
conceitos errados
1/ Perguntas frequentes do
proprietário
A) Com que frequência devo
vermifugar meu cão?
© Valérie Freiche
Muitos proprietários não têm consciência dos riscos
acarretados por parasitas intestinais à saúde de seu
animal (e, na verdade, a eles mesmos), considerando
que, se não há sinais de parasitas nas fezes do animal,
então não há necessidade de administrar antihelmínticos (Figura 1).
O outro problema
frequentemente enfrentado é que o proprietário utiliza
seus próprios vermífugos, o que é ineficaz contra certos
parasitas ou exige a repetição do tratamento para ser
eficaz. O intervalo entre a vermifugação dos cães
variará, dependendo do estilo de vida do animal — um
cão de caça que procura diariamente por forragem em
um clima quente necessitará de vermifugação mensal,
enquanto um animal pequeno que, raramente, sai de
casa talvez necessite de vermifugação semestral (ou
seja, 2 vezes ao ano), a menos que outros fatores (p. ex.,
a presença de vetores de parasitas, como pulgas)
estejam em jogo. Em todos os casos, a prescrição de
vermífugo tipo universal em uma única dose e de fácil
administração incentivará a obediência do proprietário
Figura 1.
Ascaris na
porção
proximal do
duodeno em
filhote de
cão.
ao tratamento, aliada à orientação satisfatória do cliente
quanto à frequência recomendada de vermifugação.
Pode ser desejável a realização de análise fecal em um
esquema periódico (1 ou 2 vezes/ano em climas mais
quentes), independentemente de o cão exibir ou não
fezes anormais.
B) Meu cão ficará doente pelo resto
da vida? Quanto tempo ele ficará
dessa forma?
Muitos distúrbios gastrentéricos crônicos infelizmente
são problemas de longo prazo — isso não significa
necessariamente que o pet sempre ficará doente ou terá
sinais clínicos, como diarreia, mas sim que ele(a) estará
sob risco de adoecer outra vez no futuro. Isso é
particularmente provável se o esquema terapêutico for
prescrito, mas não adotado ou, (talvez mais
provavelmente), o proprietário perceba que, tendo
iniciado o tratamento, o cão está melhor e interrompa a
medicação. A boa notícia é que muitos distúrbios podem
ser tratados com êxito, permitindo que o pet tenha uma
vida plena e feliz, com mínimo desconforto ou
inconveniência; é preciso que isso seja persuasivamente
comunicado ao proprietário.
C) Qual será o custo do diagnóstico
e tratamento?
Os custos de tratamento da doença variam
enormemente, dependendo do diagnóstico. À medida
que a medicina veterinária se desenvolve alinhada com
o progresso na medicina humana, é possível tratar com
sucesso muitos distúrbios veterinários, que eram
incuráveis há alguns anos. Contudo, isso pode incorrer
em um preço - particularmente pelo fato de muitos
51
Focus_SPECIAL_GI_port:01focuschatp0108FR 12/05/11 15:37 Página 52
A r m a d i l h a s
e m
d i s t ú r b i o s
distúrbios gastrentéricos serem de natureza crônica e
pela possibilidade de recorrência se o tratamento for
interrompido ou reduzido. É útil fornecer aos
proprietários uma estimativa precisa de gastos (tanto
em termos de avaliações iniciais como dos possíveis
custos contínuos) o mais cedo possível no planejamento
terapêutico, fazendo com que eles ponderem o
orçamento da doença de seu animal ou questionem a
possibilidade de opções terapêuticas mais baratas
antes de uma abordagem potencialmente dispendiosa
ser adotada.
g a s t r i n t e s t i n a i s
n o
c ã o
Os distúrbios gastrintestinais geralmente podem ser
tratados sem comprometer muito o estilo de vida do cão
ou do proprietário; além disso, deve-se ressaltar que,
quanto mais preparado estiver o proprietário para
desempenhar um papel ativo no tratamento, menor será
a probabilidade de surgimento de problemas no futuro.
Muitas vezes, o tratamento de problemas GI a longo
prazo não deve ser fastidioso ou pesado, mas assegurar
que o cão seja alimentado com uma dieta fixa todo dia e
não tenha acesso a petiscos ou garantir que o animal
receba a dose correta da medicação no(s) horário(s)
recomendado(s) todo dia. Em alguns casos em que a
obediência do cliente ao tratamento pareça improvável,
a ponto de comprometer o êxito do esquema
terapêutico, é essencial que o clínico gaste algum
tempo, abordando as exigências do tratamento com o
proprietário e explorando os possíveis pontos de tropeço
e possíveis soluções antes de embarcar em um curso
terapêutico selecionado.
D) A medicação a longo prazo será
perigosa para meu cão?
Os proprietários frequentemente se preocupam com a
aparente probabilidade de que seu cão necessite de
tratamento a longo prazo para algum distúrbio. Todos os
medicamentos com licença de uso em animais
domésticos são extensivamente testados em termos de
segurança e eficácia antes de serem aprovados. Sabese que até mesmo os medicamentos que não possuem
licença para uso veterinário — como muitos agentes
terapêuticos de seres humanos — são bastante
seguros em animais. Contudo, é importante que os
proprietários tenham consciência dos possíveis efeitos
colaterais de qualquer medicação sugerida pelo
veterinário (especialmente por período de tempo
prolongado) em qualquer animal, juntamente com a
tranquilização de que as reavaliações regulares do
animal (aliadas com exames laboratoriais, conforme a
necessidade) ajudarão a garantir que quaisquer
problemas relacionados ao medicamento sejam
minimizados, mas identificados se aparecerem. Isso
pode ajudar a melhorar a obediência do cliente ao
tratamento — pode ser muito frustrante para o
veterinário se deparar com a recidiva de algum quadro,
porque o proprietário interrompeu a medicação,
possivelmente porque ele(a) estava preocupado com os
efeitos colaterais. Deve-se enfatizar que, em casos GI
crônicos, é extremamente improvável que o tratamento
seja pior que a doença.
F) Posso fornecer ao meu animal de
estimação qualquer outro alimento
além da dieta prescrita se ele estiver
sofrendo de doença inflamatória
intestinal?
A doença inflamatória intestinal é uma doença
complexa. O tratamento prescrito é bastante variável,
dependendo do diagnóstico exato da causa dessa
doença inflamatória intestinal; no entanto, se a dieta foi
identificada como um recurso que ajuda o cão a lidar
com o problema, é muito melhor não arriscar na troca do
alimento fornecido. A variedade de opções alimentares
apreciadas pelos seres humanos não reflete
necessariamente nos cães — para muitos animais,
importa o fato de eles serem alimentados, não com o
quê eles são alimentados; dessa forma, é importante
que os proprietários consigam resistir à tentação de
alimentar seu animal com algo diferente “só para variar
um pouco”. Em casos de doença inflamatória intestinal,
é particularmente relevante enfatizar que a introdução
de uma nova fonte de alimento pode deflagrar a
recorrência de sinais clínicos — sempre é
recomendável a busca por orientação veterinária, mas,
E) O que preciso fazer – como
proprietário – para tratar meu cão?
52
Focus_SPECIAL_GI_port:01focuschatp0108FR 12/05/11 15:37 Página 53
A r m a d i l h a s
e m
d i s t ú r b i o s
g a s t r i n t e s t i n a i s
n o
c ã o
surgir qualquer preocupação, sempre se deve buscar por
orientação médica.
em geral, talvez seja melhor a adesão estrita a uma
dieta recomendada se ela agradar ao animal, evitandose quaisquer petiscos ou gêneros alimentícios
alternativos.
I) Você consegue remover o linfoma
de meu cão?
G) A comida caseira é melhor que
as rações comerciais ou dietas de
prescrição?
O linfoma é um tumor maligno, que se desenvolve
dentro do sistema linfático do animal. Além de ser um
sistema importante do corpo, o sistema linfático
consiste na interconexão de vasos linfáticos e
linfonodos, que se estendem por todo o corpo do animal;
no entanto, isso também torna muito fácil a
disseminação de qualquer câncer do sistema linfático
por todo o organismo. Portanto, se, por exemplo, a
cirurgia for realizada na perna do cão para remover
alguma glândula linfática que contenha um linfoma, o
câncer quase seguramente se disseminará para outro
lugar do corpo, mesmo se ainda não estiver aparente.
Nesses casos, é muito melhor considerar o uso de
tratamentos médicos, como agentes quimioterápicos,
que frequentemente podem ser muito bem-sucedidos
no tratamento desse tipo de tumor.
A nutrição é uma ciência complexa. Os fabricantes de
pet food gastam enormes quantias em dinheiro
pesquisando o que torna uma dieta ideal para cães e
gatos — e, certamente, a resposta varia, dependendo
do tipo de animal que está sendo considerado. A melhor
dieta para cão de trabalho não é a mesma que aquela
para cão doméstico de pequeno porte. Do mesmo modo,
é mais provável que as dietas de prescrição
desenvolvidas para animais acometidos por doença
responsiva à dieta ajudem a obter uma dieta balanceada
ideal para o pet do que a comida caseira aleatória (às
vezes bem-sucedida, outras vezes não), o que quase
com certeza não será tão bem-balanceada quanto à
dieta comercial em termos de vitaminas, minerais e
nutrientes. Na grande maioria das vezes os alimentos
industrializados oferecem melhores respostas que as
dietas caseiras.
J) Por que o outro veterinário não
diagnosticou isso?
H) Há riscos para outros
animais/pessoas da casa
(especialmente crianças)?
Esta é uma pergunta frequentemente feita pelos
proprietários que visitam uma clínica de
encaminhamento ou buscam uma segunda opinião de
outro veterinário. É importante compreender que, muitas
vezes, nos estágios iniciais de uma doença, pode ser
difícil ou até mesmo impossível afirmar com certeza que
um animal tem algum problema específico — talvez um
crescimento não seja grande o suficiente para ser
notado à radiografia ou uma série inicial de amostras de
sangue não revelou qualquer anormalidade. Em outros
casos, pode ser que o segundo veterinário tenha acesso
a equipamentos mais sofisticados, o que auxilia no
diagnóstico, ou tenha conhecimento especializado em
uma área em particular, o que faz com que ele(a) se sinta
confortável em um diagnóstico fácil, quando outros
clínicos gerais não estarão familiarizados com algumas
das doenças mais raras; certamente, esse
conhecimento especializado e/ou equipamentos
diagnósticos exclusivos são duas das principais razões
Muitas doenças que acometem pequenos animais não
são capazes de afetar os seres humanos — qualquer
distúrbio que tenha o potencial de passar dos animais
para os seres humanos recebe o nome de zoonose. Em
distúrbios GI, as principais doenças zoonóticas são as
infecções bacterianas, como Salmonella e
Campylobacter. Alguns parasitas, como Toxocara e
Giardia, também conseguem passar dos cães para os
seres humanos. É prudente, sempre que for tratar um
animal, tomar precauções básicas de higiene; no
entanto, nos casos em que o animal foi diagnosticado
com algum distúrbio com potencial de infectar os seres
humanos, é sábio ser muito rigoroso para garantir que o
risco de transmissão de doença seja minimizado e, se
53
Focus_SPECIAL_GI_port:01focuschatp0108FR 12/05/11 15:37 Página 54
A r m a d i l h a s
e m
d i s t ú r b i o s
para o encaminhamento de casos difíceis a veterinários
focados em áreas específicas da medicina veterinária,
como a gastrenterologia.
g a s t r i n t e s t i n a i s
n o
c ã o
Algumas vezes, pode ser difícil dar um prognóstico
preciso nos estágios iniciais de uma doença ou nos
casos em que o diagnóstico é apenas possível, mas não
definitivo. Existem algumas condições em que o
desfecho pode variar muito, dependendo do número
total de fatores envolvidos. É um clínico prudente aquele
que explica ao proprietário do animal de estimação o
fato de que a natureza de alguns distúrbios GI torna
difícil ou impossível predizer como seu animal estará em
6 semanas, 6 meses ou, até mesmo, 6 anos com
determinado problema. Contudo, a maioria dos
proprietários apreciará uma avaliação honesta, mesmo
se o veterinário puder lhes fornecer apenas as chances
(em porcentagem) de possíveis desfechos ou lhes
advertir das possíveis falhas ou problemas associados.
K) Tenho medo de quimioterapia
Muitos proprietários fogem da palavra “quimioterapia”
por causa do que eles sabem — ou algumas vezes
pensam que sabem — sobre tratamento quimioterápico
em seres humanos e seus graves efeitos colaterais. Em
geral, o uso de agentes para tratamento de neoplasias
malignas em cães exerce efeitos colaterais em menor
quantidade e gravidade — em parte porque, em seres
humanos, esses medicamentos são utilizados em uma
dosagem máxima na tentativa de erradicar o tumor. Com
os pets, a situação tende a ser um tanto diferente. Os
veterinários tendem a buscar pela remissão do tumor,
embora ainda possibilitem que o animal goze de boa
qualidade de vida, com níveis mínimos dos efeitos
deletérios e do sofrimento frequentemente observados
em seres humanos. Em geral, os agentes
quimioterápicos podem aumentar muito a qualidade de
vida do pet e permitir um prolongamento significativo de
seu tempo de vida, sem causar problemas como
alopecia e supressão da medula óssea, mas é essencial
explicar isso ao proprietário ao se considerar as opções
terapêuticas para neoplasias. No entanto, também é
prudente conhecer a potência e as possíveis
complicações decorrentes do uso desses medicamentos
e utilizá-los com respeito ao bem-estar do homem e do
animal; a avaliação regular da saúde do paciente é
fundamental e, dessa forma, o uso dos quimioterápicos
nunca deve ser levianamente empreendido.
M) Meu cão está constipado - o
que devo fazer?
A constipação é, por definição, um distúrbio em que os
movimentos intestinais são muito infrequentes e, com
isso, as fezes ficam duras e secas. Não há motivo para
se preocupar se o cão ocasionalmente não defeca todos
os dias. Entretanto, quando a constipação se torna
crônica e, na ausência de quaisquer outros sinais
clínicos, é aconselhável enriquecer a dieta do animal
com fibras, garantir atividade física diária apropriada e
reduzir qualquer peso em excesso.
Em alguns casos de constipação, todavia, certos
critérios podem justificar a intervenção médica:
• defecação dolorosa
• presença de sangue nas fezes ou sangue após a
defecação
• início súbito de constipação nos casos em que não há
problema prévio
• presença de tenesmo (esforço para defecar).
L) Meu cão vai sobreviver?
> Perguntas a Denise Elliott
Por quanto tempo devemos prescrever uma dieta em caso de doença GI nos cães?
?
O período de tempo de prescrição de alguma dieta para o tratamento de doença gastrintestinal em cães depende muito
mais da fisiopatologia ou etiologia subjacente. Os cães com gastrenterite aguda talvez necessitem de terapia dietética
gastrintestinal específica por apenas 1 semana, enquanto aqueles com distúrbios de motilidade ou doença inflamatória
intestinal necessitarão de tratamento nutricional pelo resto da vida.
Denise Elliott
54
Focus_SPECIAL_GI_port:01focuschatp0108FR 12/05/11 15:37 Página 55
e m
d i s t ú r b i o s
g a s t r i n t e s t i n a i s
n o
c ã o
© Valérie Freiche
A r m a d i l h a s
Figura 2a. Corpo estranho
ósseo (vértebra de ovelha)
preso na porção distal do
esôfago de cão macho da raça
West Highland White Terrier
(Terrier Branco das Colinas do
Oeste) de 2 anos de idade.
Figura 2b. Lesões parietais
ulcerativas observadas em
posição proximal ao cárdia,
causadas pelo atrito de osso
contra a parede do esôfago.
Figura 2c. Cicatrização
parietal observada 4 dias mais
tarde, após tratamento
médico.
recomendado por boa parte dos veterinários.
Essas situações justificam a tentativa de averiguar a
etiologia por meio de vários exames, especialmente a
colonoscopia.
2/ Perguntas frequentes do
veterinário
N) Posso dar ossos a meu cão?
A) A cirurgia é o último recurso no
exame de casos gastrentéricos? São
realizadas muitas laparotomias
exploratórias?
Alguns proprietários acreditam que não há perigo em
fornecer ossos a seus cães e têm feito isso por vários
anos sem problemas sérios. Mesmo quando a tolerância
à ingestão continua boa por um período de tempo
prolongado, as complicações sempre são possíveis e
imprevisíveis, p. ex., fragmentos de osso ou cartilagem
podem ficar presos no esôfago, causando disfagia, e
talvez necessitem de extração endoscópica ou cirúrgica
imediata e complexa (Figuras 2).
Sempre é recomendável evitar a ingestão de quantidade
excessiva de osso, pois isso pode levar a problemas com
defecação, já que restos ósseos parcialmente digeridos
podem afetar as fezes. De vez em quando, se as fezes
estiverem excessivamente duras e impossíveis de
expelir, talvez haja necessidade do emprego de
anestesia geral para permitir a remoção manual desses
fragmentos do reto. Mesmo ossos grandes, se
fornecidos aos cães, podem ocasionar problemas como
fraturas dentárias. Dessa forma, o fornecimento de
ossos a cães e gatos é bastante controverso e não é
FPara muitos clínicos gerais com acesso limitado ou nulo
a equipamentos mais sofisticados de diagnóstico por
imagem, como endoscopia ou ultrassonografia, a
técnica de laparotomia exploratória é atrativa. Apesar
de sua natureza invasiva, a laparotomia tem o benefício
de conferir excelente visualização das vísceras
abdominais, podendo possibilitar o diagnóstico imediato
e definitivo (e, possivelmente, o tratamento) de alguma
doença e permitir a realização de amostragem direta de
órgãos. Contudo, é importante enfatizar que esse
procedimento não tem utilidade para o diagnóstico de
muitos problemas, tais como distúrbios de motilidade e
síndromes de má-absorção e, quando utilizado sem
critério ou prudência, pode ser um substituto
insatisfatório para técnicas investigativas mais
sofisticadas e menos invasivas. Idealmente, o clínico
55
Focus_SPECIAL_GI_port:01focuschatp0108FR 12/05/11 15:37 Página 56
A r m a d i l h a s
e m
d i s t ú r b i o s
g a s t r i n t e s t i n a i s
c ã o
existe um método correto ou incorreto de tratar esse tipo
de doença inflamatória intestinal; as escolhas
terapêuticas dependerão de vários fatores, ou seja,
como o cão responde ao tratamento inicial selecionado
(p. ex., dieta de prescrição ou corticosteroides), se o
proprietário é capaz de lidar com o tratamento, qual o
grau de comprometimento do cão, e se há ou não
fatores de complicação.
deve ter uma boa ideia do que ele está procurando antes
de iniciar a cirurgia, mas a laparotomia sempre deve ser
feita de forma metódica e com diligência. Também é
preciso notar que as amostras de biopsia sempre devem
ser obtidas mesmo em caso de anatomia macroscópica
normal, conforme exposto a seguir:
• Caso se observe diarreia, devem-se fazer 3 biopsias
do intestino delgado (duodeno, íleo, jejuno), somadas a
biopsias de linfonodo (se estiver infartado) e fígado.
• Caso se observe vômito, devem-se realizar biopsias do
estômago e do pâncreas, bem como das amostras
citadas anteriormente.
É aconselhável a realização de biopsias de espessura
completa do intestino grosso apenas se houver sinais
significativos de doença nesse segmento intestinal.
C) Qual é a utilidade da citologia
via retal?
A citologia retal é um método simples, mas subusado,
de pesquisa de problemas GI; durante o exame por via
retal, o clínico pode usar o dedo para esfoliar algumas
células do revestimento retal e empregar uma coloração
adequada para permitir o exame microscópico. Embora,
em muitos casos, não se observem anormalidades,
achados como grande número de neutrófilos ou
© Valérie Freiche - Clinique Frégis
B) Qual o grau de importância dos
quadros de doença inflamatória
intestinal/síndrome do intestino
irritável em cães? Qual é a melhor
forma de tratar a doença
inflamatória intestinal?
A síndrome do intestino irritável e a doença inflamatória
intestinal são frequentemente confundidas nas mentes
tanto dos veterinários como dos proprietários. A
expressão “síndrome do intestino irritável” atravessou a
medicina humana, mas acredita-se que ela seja o
resultado de vários tipos de estresses; essa doença
pode se manifestar sob a forma de diarreia recorrente e
intermitente, sem lesões intestinais patológicas. A
doença inflamatória intestinal é um termo coletivo para
uma variedade de distúrbios infiltrativos do intestino
delgado. Deve-se enfatizar que, por si só, a doença
inflamatória intestinal é simplesmente uma descrição,
não um diagnóstico definitivo. Na suspeita de doença
inflamatória intestinal, é sensato tentar definir o que
causou a inflamação crônica do intestino via biopsia
intestinal. Acredita-se que a doença inflamatória
intestinal seja muito mais disseminada em cães do que
se imaginou pela primeira vez; contudo, deve-se resistir
à tentação de rotular qualquer caso com sinais
gastrentéricos crônicos como “doença inflamatória
intestinal” sem avaliação diagnóstica minuciosa. Não
n o
Figura 3. Os valores de eletrólitos são
inestimáveis para casos agudos,
especialmente nos casos em que há
necessidade de fluidoterapia.
56
Focus_SPECIAL_GI_port:01focuschatp0108FR 12/05/11 15:37 Página 57
A r m a d i l h a s
e m
d i s t ú r b i o s
linfócitos podem ser muito úteis para sugerir o
diagnóstico; no entanto, talvez haja necessidade de
técnicas de biopsia definitivas para fornecer um
diagnóstico conclusivo.
g a s t r i n t e s t i n a i s
n o
c ã o
E) Quais os exames de sangue mais
úteis?
Os exames de bioquímica sérica e hematologia podem
ser bastante proveitosos para auxiliar na avaliação de
problemas GI; em particular, esses exames geralmente
ajudarão a descartar ou confirmar a presença de doença
sistêmica. Os valores de eletrólitos são inestimáveis
para casos agudos, especialmente nos casos em que há
necessidade de fluidoterapia (Figure 3). Podem ser
indicados exames mais específicos, dependendo do
histórico, do exame clínico e dos resultados de outros
métodos diagnósticos. Apenas se pode enfatizar que os
exames para doenças como insuficiência pancreática
exócrina e supercrescimento bacteriano do intestino
delgado são específicos para esses distúrbios e não
necessariamente parte da avaliação diagnóstica inicial
de qualquer caso. Além disso, é preciso ressaltar que a
interpretação de todos os exames de sangue deve ser
feita em relação a cada caso individual e ao bom
conhecimento de variantes normais.
D) Quais os exames fecais mais
úteis?
Para muitos veterinários, o uso de exame de fezes
começará e terminará com a pesquisa de parasitas. Isso,
certamente, é inestimável nos casos em que
microrganismos como Giardia, Toxoplasma e
Cryptosporidia estão presentes. Outra vez, é essencial o
conhecimento satisfatório da confiabilidade desses
exames — a liberação intermitente de alguns parasitas
(p. ex., Giardia) pode tornar inútil um teste negativo. A
coprocultura tem utilidade apenas nos casos em que se
identificam patógenos bacterianos conhecidos como
Campylobacter e Salmonella. Muitas vezes, os clínicos
superinterpretam o relatório bacteriológico e assumem
que a microbiota intestinal normal relatada (p. ex., E.
coli) é patogênica, iniciando antibioticoterapia
inapropriada. Outros exames de fezes, como exame em
busca de grânulos de amido não digeridos, fibras
musculares, glóbulos de gordura e sangue oculto, talvez
sejam proveitosos, mas não costumam ser, por si só,
diagnósticos em doença GI.
F) Qual a utilidade da radiografia
em casos GI? O exame radiográfico
com bário ainda é importante?
A radiografia simples é extremamente útil na pesquisa
> Perguntas a Denise Elliott
Qual o interesse de dietas hidrolisadas no tratamento de doença inflamatória intestinal em cães?
Denise Elliott
Existem, pelo menos, três fatores críticos envolvidos na patogênese da doença inflamatória intestinal; ruptura da barreira
da mucosa, alterações da microbiota bacteriana gastrintestinal e desregulação da resposta imune da mucosa.
A antigenicidade das proteínas da dieta pode ser minimizada por hidrólise enzimática até produzir hidrolisados proteicos
de baixo peso molecular. A diminuição do tamanho das proteínas ingeridas reduz a probabilidade de ligação cruzada com
imunoglobulinas e a subsequente degranulação de mastócitos.
As proteínas incompleta ou insatisfatoriamente digeridas têm maior potencial de incitar uma resposta imune às proteínas
antigênicas residuais e grandes polipeptídeos. Por outro lado, as proteínas de alta digestibilidade são completamente
digeridas em aminoácidos livres e pequenos peptídeos, que possuem menor potencial de iniciar uma resposta alérgica. As
dietas à base de proteínas hidrolisadas são muito digeríveis, com digestibilidades proteicas superiores a 92%. Uma dieta
de alta digestibilidade requer menor quantidade de secreções gástricas, pancreáticas, biliares e intestinais para digestão.
Isso resulta em digestão e absorção quase completa no intestino delgado superior, de modo que uma quantidade mínima
de resíduo é apresentada à porção inferior do intestino. Resíduos mínimos diminuem os subprodutos que podem contribuir
para os processos de inflamação, diarreia osmótica ou resposta imune anormal.
?
57
Focus_SPECIAL_GI_port:01focuschatp0108FR 12/05/11 15:37 Página 58
A r m a d i l h a s
e m
d i s t ú r b i o s
g a s t r i n t e s t i n a i s
n o
c ã o
perfil de sensibilidade?
de alguns distúrbios GI. A natureza não invasiva torna o
exame radiográfico uma ferramenta proveitosa para o
diagnóstico de problemas como corpos estranhos
gastrintestinais; no entanto, caso sejam obtidas
radiografias normais, esse exame pode não permitir a
exclusão de objetos radiotransparentes. As técnicas de
bário são tradicionalmente utilizadas em conjunto com
radiografia simples para enfrentar esse problema. Os
estudos com contraste ainda são relevantes, pois não se
pode garantir que outros métodos como endoscopia
identifiquem objetos estranhos ou problemas
intraluminais em todos os níveis do trato GI. O bário
ainda é de enorme uso na pesquisa de distúrbios de
motilidade em combinação com a fluoroscopia. O
principal problema enfrentado por estudos com
contraste é que os clínicos muitas vezes não estão
familiarizados com a técnica para empregá-la de forma
apropriada ou frequentemente utilizam quantidade/tipo
de agente de contraste inadequado e colocam o
diagnóstico em risco.
Pode ser muito tentador tratar qualquer resultado
microbiológico recebido de algum laboratório como um
diagnóstico — ou seja, se uma bactéria em particular foi
identificada, ela deve ser a causa do distúrbio GI e
tratada de acordo com o perfil de sensibilidade. Essa
abordagem pode ser incorreta, principalmente pelo fato
de que muitas bactérias isoladas na coprocultura são
pertencentes à microbiota intestinal normal; além disso,
o uso de antibióticos alterará o equilíbrio dessa
microbiota, possivelmente em detrimento do animal (ver
adiante). Caso se encontre uma infecção específica,
como Salmonella, Campylobacter ou Giardia, é
totalmente correto tratar com algum agente apropriado.
Em caso de dúvida, o contato com o laboratório pode ser
muito útil para decidir se o microrganismo isolado é ou
não clinicamente significativo.
I) Os antibióticos devem ser
utilizados apenas em caso de
isolamento de bactérias
significativas?
G) Os quadros de insuficiência
pancreática exócrina e
supercrescimento bacteriano do
intestino delgado são comuns em
cães?
Geralmente, os veterinários superprescrevem
antibióticos para distúrbios GI. Isso pode ocorrer por
causa da crença mal-colocada de que muitos cães com
diarreia apresentam infecção bacteriana ou porque o
proprietário está ávido ou ansioso pela prescrição dessa
classe de medicamentos. É recomendável resistir à
tentação de prescrever um antibiótico em base empírica;
isso não apenas pode simplesmente permitir a evolução
da doença ainda mais sem exames/tratamentos
adequados, mas o uso de antibióticos inapropriados
pode, na verdade, agravar alguns problemas GI — como
nos casos em que os antibióticos induzem ao vômito ou
alteram a microbiota microbacteriana normal. Em geral,
os antibióticos devem ser utilizados somente quando há
provas satisfatórias de que esses agentes terão efeitos
benéficos sobre o paciente (isso, por sua vez, requer a
tentativa de obtenção do diagnóstico definitivo pelo
clínico antes de começar o tratamento), a menos que
haja indícios de outros problemas, como leucopenia,
neutrofilia ou choque, em que os antibióticos podem ser
satisfatoriamente benéficos como terapia primária
Tanto a insuficiência pancreática exócrina como o
supercrescimento bacteriano do intestino delgado
devem ser consideradas no diagnóstico diferencial de
diarreia crônica em cães e, portanto, é recomendável a
inclusão de exames apropriados em qualquer avaliação
diagnóstica de tais casos. Deve-se notar que muitos
casos de supercrescimento bacteriano do intestino
delgado podem ser secundários a algum outro distúrbio
entérico, mais do que uma doença por si só; por
exemplo, acredita-se que a insuficiência pancreática
exócrina possa predispor ao supercrescimento
bacteriano do intestino delgado em alguns casos. A
interpretação dos resultados dos exames como
estimativas de folato/cobalamina deve ser feita com
cuidado, já que muitos casos de diarreia crônica podem
ser de origem multifatorial.
H) Como devo interpretar os
resultados da bacteriologia e o
58
Focus_SPECIAL_GI_port:01focuschatp0108FR 12/05/11 15:37 Página 59
A r m a d i l h a s
e m
d i s t ú r b i o s
g a s t r i n t e s t i n a i s
n o
c ã o
é a primeira vez que seu animal sofre desse problema ou
de quadro semelhante. Pode ser que o proprietário nem
sempre se dê conta de que episódios isolados de doença
por vários meses, com manifestações possivelmente
distintas de sinais clínicos, podem ser atribuídos ao
mesmo problema inicial
antes de se obter qualquer diagnóstico definitivo.
J) Com que rapidez o veterinário
deve reavaliar os casos se não
houver resposta ao tratamento?
B) Os corticosteroides são perigosos
para os animais?
A reavaliação de qualquer paciente depende
principalmente do diagnóstico presuntivo, mas é
essencial que o clínico tenha conhecimento do provável
tempo de resposta para o diagnóstico apropriado,
recorrendo, se necessário, a livros de referência. É
potencialmente deletério para o paciente se o clínico
não estiver disposto a rever o diagnóstico caso o animal
não melhore como previsto. Em geral, caso se acredite
que a doença seja de natureza aguda, ela deve se
resolver rapidamente (e, dessa forma, o caso deve ser
reavaliado com rapidez se nenhuma melhora for
observada); alternativamente, um distúrbio crônico pode
levar algumas semanas para responder ao tratamento.
Em casos de supercrescimento bacteriano do intestino
delgado, talvez haja necessidade de 4 semanas de
antibióticos para se observar uma resposta satisfatória;
para problemas relacionados à dieta, talvez seja
necessário aguardar 6 semanas antes de decidir sobre a
eficácia do tratamento. Em todos os casos em que a
resposta é pior do que a prevista, é essencial que o
clínico tenha certeza de que o proprietário está seguindo
o esquema terapêutico corretamente.
O uso disseminado e, algumas vezes, imprudente de
glicocorticoides tanto em animais como em seres
humanos ao longo dos anos contribuiu para a frequente
crença de que esse grupo de medicamentos seja
extremamente perigoso. Embora seja inegável que o uso
de corticosteroides a longo prazo possa produzir sérios
efeitos adversos sobre o paciente, a utilidade desses
agentes é tão importante que eles devem ser prescritos
quando apropriados. O emprego de corticosteroides é
idealmente limitado para casos em que o diagnóstico
definitivo foi estabelecido, em vez de usá-los em uma
base mais empírica. Nos distúrbios gastrintestinais em
que os corticosteroides parecem ser o medicamento de
escolha, o clínico deve selecionar o glicocorticoide mais
adequado para o paciente, calculando-se a dose ideal
necessária para conferir os efeitos benéficos do
corticosteroide, ao mesmo tempo em que se minimizam
os possíveis efeitos colaterais indesejáveis. Com o
cálculo preciso da dose, o uso de terapia em dias
alternados (quando praticável e prudente) e os períodos
de reavaliação (aliados com exames propícios para
avaliar quaisquer efeitos adversos do corticosteroide),
deve ser possível garantir que o medicamento trate com
êxito o distúrbio médico sem gerar consequências
deletérias.
3/ Conceitos errados na
mente do veterinário
A) Quando a doença é crônica?
C) É normal ingerir grama e outras
plantas? Qual o significado da
postura comportamental anormal?
Com frequência, os clínicos dividem a doença
gastrintestinal em aguda ou crônica, pois a avaliação, o
diagnóstico diferencial e o tratamento podem variar
acentuadamente entre as duas categorias. É aceito que
qualquer distúrbio presente por mais de 2 a 3 semanas
se classifica como um problema crônico; no entanto, é
importante lembrar que não é raro que uma doença seja
intermitente em sua manifestação e, dessa forma, o
clínico sempre deve averiguar com o proprietário se esta
Independentemente da idade ou raça, os cães muitas
vezes se mostram não seletivos na escolha de seus
alimentos, conforme evidenciado pela frequente
ingestão de uma variedade de corpos estranhos, como
ossos, material plástico, pedras, tecidos, etc. Os
59
Focus_SPECIAL_GI_port:01focuschatp0108FR 12/05/11 15:37 Página 60
A r m a d i l h a s
e m
d i s t ú r b i o s
proprietários tendem até mesmo a considerar normal
que seu animal induza seu vômito quando sai à rua; o
veterinário deve estar ciente de que, ao coletar o
histórico do caso, o vômito resultante pode não ser
relatado pelo proprietário, pois ele o considera
insignificante. A ingestão demasiada ou regular de
vegetação (capim ou outras plantas) e até mesmo o
hábito de pica (p. ex., a lambedura de ladrilhos, a
ingestão de pedras, etc.) sempre devem ser
considerados como patológicos, em particular se ocorrer
várias vezes por semana. Esse comportamento indica,
na grande maioria dos casos, dor gastroduodenal.
O mesmo é verdadeiro com a descrição típica de um cão
que começa a adotar posturas ou atitudes corporais
anormais que, algumas vezes, podem ser os únicos
sintomas de lesões digestivas, por exemplo:
• Se o animal começar a dormir em áreas não habituais
ou mudar de posição em que ele dorme (p. ex., em
decúbito dorsal ou, alternativamente, em decúbito
esternal com o estômago em contato com alguma
superfície fria).
• Se o animal adotar uma posição atípica, como
“postura de prece” (patas dianteiras flexionadas, cabeça
para trás, patas traseiras tensas), ou demonstrar
agitação, ou exibir mordedura dos flancos, etc.
Em suma, é importante pesquisar esses sinais clínicos
durante a consulta e lhes dar o valor diagnóstico que
eles merecem, sem assumir imediatamente que um
problema comportamental seja a causa primária desses
sinais.
g a s t r i n t e s t i n a i s
n o
c ã o
ataque
viral
grave,
síndrome
ou
peritonite.
obstrutiva/oclusiva
• A presença de alimento não digerido por
longo período de tempo após as refeições
sugere síndrome de retenção gástrica de
origem funcional (gastroparesia) ou anatômica
(disfunção pilórica, gastropatia hipertrófica,
tumor pilórico ou duodenal proximal, corpo
estranho gastroduodenal, etc.) ou pancreatite.
• O vômito que ocorre pela manhã com o
estômago vazio pode ser atribuído a refluxo
duodenogástrico de bile, particularmente em
raças de pequeno porte (Bichon Frisé, Poodle,
Yorkshire
Terrier,
Chihuahua,
etc.).
• Um grande volume de vômito é
frequentemente produzido na presença de
síndrome obstrutiva/oclusiva ou retardo do
esvaziamento
gástrico.
Entretanto,
também
existem
muitas
“armadilhas
diagnósticas”:
• Os proprietários podem achar difícil
distinguir entre regurgitação tardia (ou seja,
alimento regurgitado um pouco depois da
ingestão) e vômito precoce (ou seja, vômito
logo após a ingestão), o que pode levar o
clínico a seguir uma sequência incorreta de
exames complementares pela coleta de
histórico
enganoso.
• A presença de sangue no vômito pode sugerir
algum distúrbio subjacente com prognóstico
muito mau (tipicamente nos casos em que uma
infiltração neoplásica está ligada com
ulceração gástrica). No entanto, o sangue
também pode ser observado em distúrbios
inflamatórios, que são completamente
benignos, por exemplo, em casos de infiltração
linfoplasmocitária gastroduodenal crônica.
• Alguns distúrbios neoplásicos apresentam
uma sintomatologia inespecífica de início
tardio. Portanto, o carcinoma gástrico em cães
exibe um quadro clínico que raramente é
patognomônico; os sinais podem incluir
disorexia, ptialismo (que pode ser de natureza
isolada) e vômito não relacionado ao alimento,
que frequentemente não contém sangue
natural nos estágios iniciais da doença. Essa
falta de especificidade torna o diagnóstico
D) Quanto ao valor diagnóstico do
vômito, as características desse
sintoma ajudam a sugerir a causa
do problema?
Ao contrário do que frequentemente se fala, o
aspecto físico do vômito ou seu tempo
inesperado de chegada em relação à hora em
que o animal comeu nem sempre é
característico do distúrbio envolvido.
Para tanto, existem critérios confiáveis como:
• O vômito incontrolável ou inesperado que
ocorre em qualquer hora durante o dia é
sugestivo de ataque metabólico, pancreatite,
60
Focus_SPECIAL_GI_port:01focuschatp0108FR 12/05/11 15:37 Página 61
A r m a d i l h a s
e m
d i s t ú r b i o s
difícil; muitas vezes, o diagnóstico definitivo é
obtido apenas em um estágio tardio da doença,
quando as possibilidades terapêuticas se
encontram reduzidas e a expectativa de vida do
animal se mostra muito curta.
• Além de serem de difícil caracterização, os
distúrbios suboclusivos crônicos podem
representar um desafio diagnóstico real ao
clínico. Por exemplo, alguns animais
apresentam
sintomas
episódicos
que
normalmente não levariam à suspeita de corpo
estranho; os sinais podem incluir disorexia,
vômito não relacionado ao alimento, diarreia
intermitente do intestino delgado e estado
geral oscilante, aliados com fases alternadas
de desânimo e atitude normal.
Em suma, o veterinário deve se esforçar para
refinar a abordagem diagnóstica, identificando
os critérios confiáveis e sugerindo a sequência
mais apropriada de exames complementares
para todo e cada caso específico.
61
g a s t r i n t e s t i n a i s
n o
c ã o
Focus_SPECIAL_GI_port:01focuschatp0108FR 12/05/11 15:37 Página 62
A r m a d i l h a s
e m
d i s t ú r b i o s
g a s t r i n t e s t i n a i s
n o
c ã o
Referências
Tams TR. Gastrointestinal symptoms. In Tams TR, ed. Handbook of
Capítulo 1
Small Animal Gastroenterology. 2.ed. St. Louis: Saunders, 2003; 1-50.
DeNovo RC, Jenkins CC. Diseases of the stomach. In: Canine Medicine
and Therapeutics, ed. N. Gorman, 1998 4.ed. Blackwell Science.
Capítulo 3
Hall EJ. Introduction to investigating gastrointestinal disease. In:
Manual of Canine and Feline Gastroenterology 2005 2.ed. BSAVA.
Bush BM. Colitis en el perro. In Trastornos gastrointestinales en perros
y gatos. Hill’s Pet Nutrition Inc 1997; 35-43.
Hall EJ. Small intestinal disease. In: Canine Medicine and Therapeutics,
ed. N. Gorman, 1998 4.ed. Blackwell Science.
Guilford G. Diagnóstico de los vómitos crónicos. In Proceedings of the
19 Congreso anual AMVAC 2002; 55-58.
Radford A et al. Development, teaching and evaluation of a
consultation structure model for use in veterinary education. Veterinary
Guilford G. Pruebas de funcionalidad gastrointestinal-porqué deben
Defence Society 2002.
utilizarlas los clínicos. In Proceedings of the 19 Congreso anual AMVAC
2002; 91-95.
Melgarejo T. Novel GI diagnosis test. Charlas Purina Veterinary Division,
Capítulo 2
Alicante 2001.
Cauzinille L, Bouvy B. Communications porto-systémiques congénitales,
Shelton GD. Myasthenia gravis and disorders of neuromuscular
1ª partie: conséquences cliniques, physiopatlologie et approche
transmission. Veterinary Clinics of North America: Small Animal
thérapeutique médicale. In Prat Med Chir Comp 2003; 38: 297-304.
Practice 2002; 32: 189-207, January.
Gualtieri M. Hiatal diseases: causes, complications and treatment. In
Tams TR. Gastrointestinal symptoms. In Tams TR, ed. Handbook of
Proceedings of the 14th ECVIM-CA Congress 2004; 55-57.
Small Animal Gastroenterology. 2.ed. St. Louis: Saunders, 2003; 1-50.
Guilford G. Diagnóstico de la diarrea crónica. In Proceedings of the 19
Willard M. Hematemesis: Más difícil de lo que creíamos. In
Congreso anual AMVAC 2002; 75-79.
Proceedings of the 39 Congreso Nacional de AVEPA 2004; 111-112.
Guilford G. Obstrucción y pseudo-obstrucción: diagnóstico y tratamiento
médico. In Proceedings of the 19 Congreso anual AMVAC 2002; 65- 73.
Capítulo 4
Guilford G. Síndrome de colon irritable. In Proceedings of the 19
Congreso anual AMVAC 2002; 87-90.
Arnjeberg J. Gastric emptying time in the dog and the cat. J Am Anim
Hosp Assoc 1992; 28:77.
Jergens AE. Inflamatory bowel disease. Veterinary Clinics of North
America: Small Animal Practice March 1999; 29 (2): 501-521.
Hudson JA, Mahaffey MB. The gastrointestinal tract. In: Practical
Veterinary Ultrasound. Ed Cartree RE, Selcer BA., Hudson JA, FinnLecoindre P. Affections inflammatoires de l´oesophage: les
Bodner ST, Mahaffey MB, Johson PL, Marich KV, Williams A. Waverly
oesophagites. Le Point Vétérinaire 2004; Nº 251, Decembre, 30-33.
Company, 1995
Magne ML. Tumefacción articular y claudicación. In: Ettinger, ed.
Jones BD. Incorporating endoscopy into veterinary practice. Compend
Tratado de medicina interna veterinaria: enfermedades del perro y el
Contin Educ Pract Vet 1997; 20: 307.
gato. Traducción al castellano de la 5.ed. Philadelphia, Saunders Co,
2000 (Ed Intermédica 2002); 86-88.
62
Focus_SPECIAL_GI_port:01focuschatp0108FR 12/05/11 15:37 Página 63
A r m a d i l h a s
e m
d i s t ú r b i o s
g a s t r i n t e s t i n a i s
n o
c ã o
Referências
Konde LJ, Green PA, Pugh CR. Radiology and ultrasonography of the
Guilford W, Strombeck D. Neoplasm of the gastrointestinal tract, APUD
digestive system. In: Handbook of Small Animal Gastroenterology 2003
tumors, endocrinopathies and the gastrointestinal Tract. In: Strombeck's
Ed. Tams T.E.
Small Animal Gastroenterology, Ed. Company WS, Davis, Stonegate
publishing CO, 1996; 519-532.
Miyabayashi T, Morgan JP. Gastric emptying in the normal dog: a
contrast radiographic technique. Vet Radiol 1984; 25:187.
Marks SL, Kather EJ. Bacterial-associated diarrhea in the dog: a critical
appraisal. Vet Clin Small Anim 2003; 33: 1029-1060.
Penninck DJ, Nyland TG, Fischer PE. Ultrasonographic evaluation of
gastrointestinal diseases in small animals. Vet Radiol 1990: 31(3): 134-
Swann H, Holt D. Canine gastric adenocarcinoma and leiomyosarcoma:
141.
a retrospective study of 21 cases (1986-1999) and literature review. J
Am Anim Hosp Assoc 2002; 38:157-164.
Penninck DJ. Ultrasonography of the gastrointestinal tract. In:
Veterinary Diagnostic Ultrasound. Ed Nylan TG, Matton JS, W.B.
Saunders Company, Philadelphia 1995.
Tams TR. In: Tams TR. Handbook of Small Animal Gastroenterology.
Robertson ID, Burbidge HM. Pros and cons of barium-impregnated
Endoscopy and Laparoscopy: Indications and Instrumentation. WB
polyethylene spheres in gastrointestinal disease. In: The Veterinary
Saunders Co, Philadelphia 1996; 137-162.
Clinics of North America- Small Animal Practice: Clinical Radiology.
Watrous BJ ed, WB Saunders, Philadelphia 2000, 30:2, 449-465.
Tams TR. In: Tams TR. Handbook of Small Animal Gastroenterology.
Endoscopy and Laparoscopy: Chronic Diseases of the Small Intestine.
WB Saunders Co, Philadelphia 1996; 267-319.
Capítulo 5
DeNovo R, Bright R. Recto-anal disease. In: Textbook of Small Animal
Washabau RJ. Gastrointestinal motility disorders and gastrointestinal
prokinetic therapy. Vet Clin Small Anim 2003; 33: 1007-1028.
Internal Medicine, Ed. WB Saunders Company, Philadelphia 2000: 1257-
Westermark E, Wiberg M. Exocrine pancreatic insufficiency in dogs. Vet
1270.
Clin Small Anim 2003; 33: 1165-1179.
Estrada M, Freiche V, Dargent F. Tumeurs gastro-intestinales chez le
chien (gastro-intestinal tumours in dogs). Prat Med Chir Anim Comp
1997; 32:99-114.
63
Focus_SPECIAL_GI_port:01focuschatp0108FR 12/05/11 15:37 Página 64
A r m a d i l h a s
e m
d i s t ú r b i o s
g a s t r i n t e s t i n a i s
Anotações Pessoais
64
n o
c ã o
Focus_SPECIAL_GI_port:01focuschatp0108FR 12/05/11 15:37 Página 65
A r m a d i l h a s
e m
d i s t ú r b i o s
g a s t r i n t e s t i n a i s
Anotações Pessoais
65
n o
c ã o
Focus_SPECIAL_GI_port:01focuschatp0108FR 12/05/11 15:37 Página 66
A r m a d i l h a s
e m
d i s t ú r b i o s
g a s t r i n t e s t i n a i s
Anotações Pessoais
66
n o
c ã o
Focus_SPECIAL_GI_port:01focuschatp0108FR 12/05/11 15:37 Página 67
A r m a d i l h a s
e m
d i s t ú r b i o s
g a s t r i n t e s t i n a i s
Anotações Pessoais
67
n o
c ã o
Focus_SPECIAL_GI_port:01focuschatp0108FR 12/05/11 15:37 Página 68
A r m a d i l h a s
e m
d i s t ú r b i o s
g a s t r i n t e s t i n a i s
n o
c ã o
Este livro foi preparado com o maior cuidado, levando-se em conta as pesquisas e descobertas científicas mais
recentes. É recomendável que o leitor recorra às prescrições e instruções de medicamentos e alimentos em virtude de
sua provável mudança. Em vista da diversidade e complexidade dos casos clínicos de cães e gatos, é imperativo saber
que quaisquer intervenções terapêuticas e exames complementares descritos nesse livro não são exaustivos, ou seja,
não se esgotaram. As soluções e os tratamentos propostos não podem, sob quaisquer circunstâncias, substituir o exame
feito por um veterinário qualificado. Os editores e os autores não se responsabilizam por qualquer falha das soluções e
dos tratamentos sugeridos.
Coordenação editorial: Laurent Cathalan
Layout: Arnaud Pouzet
Gerente Técnico: Buena Media Plus
Ilustrações: Edouard Cellura
Revisão Português (Brasil): Luciana Domingues de Oliveira (Médica Veterinária)
© 2011 Royal Canin do Brasil
BP 4
www.royalcanin.com.br
Esta obra é uma tradução da Focus Special "Pitfalls in GI disorders in dog", publicada em março de 2010 pela
Royal Canin França (reimpresso da edição de 2006).
Nenhuma parte dessa publicação pode ser reproduzida sem o consentimento prévio do autor, bem como de seus sucessores ou sucessores legais,
em conformidade com a Lei de Propriedade Intelectual (Artigo I, 112-4). Qualquer reprodução parcial ou completa constitui uma falsificação passível de processo criminal. Apenas reproduções (Art. I. 122-5) ou cópias estritamente reservadas para uso privado do copiador, bem
como breves citações e análises (incluídas) justificadas pela natureza pedagógica, crítica ou informativa do livro, estão autorizadas, sujeito ao
cumprimento das provisões dos artigos L. 122-10 até L. 122-12 do Código de Propriedade Intelectual relativo à reprodução fotográfica.
68

Documentos relacionados